You are on page 1of 70

Strictly according to the latest syllabus for ICSE (Class IX)

prescribed by the Council for the Indian School Certificate Examinations, New Delhi.

S. Chand’s
ICSE
Mathematics

d
ite
m
Li
Book 1

ny
for Class
pa
IX
om
O.P. Malhotra S.K. Gupta
C

M.A. (Gold Medalist) Principal (Retd.)


Former Head of Mathematics Birla Vidya Mandir, Nainital
d

Department Former Chairman


An

The Doon School, Dehradun Indian Public Schools’ Conference


n d

Anubhuti Gangal
ha

M.A. (Gold Medalist), M.Ed.


Formerly, Senior Faculty Member
The Daly College, Indore
C

Birla Vidya Mandir, Nainital


S
©
S CHAND SCHOOL BOOKS
(An imprint of S Chand Publishing)
A Division of S Chand And Company Limited
(An ISO 9001 Certified Company)
Head Office : B-1/D-1, Ground Floor, Mohan Co-operative Industrial Estate, Mathura Road, New Delhi–110 044
Phone : 011–6667 2000, e-mail : info@schandpublishing.com
Registered Office : A-27, 2nd Floor, Mohan Co-operative Industrial Estate, New Delhi–110 044
www.schandpublishing.com; e-mail : helpdesk@schandpublishing.com

Branches :
Ahmedabad : Ph: 079-2754 2369, 2754 1965, ahmedabad@schandpublishing.com
Bengaluru : Ph: 080-2235 4008, 2226 8048, bangalore@schandpublishing.com
Bhopal : Ph: 0755-4274 723, 4209 587, bhopal@schandpublishing.com

d
Bhubaneshwar : Ph: 0674-2951 580, bhubaneshwar@schandpublishing.com

ite
Chennai : Ph: 044-2363 2120, chennai@schandpublishing.com
Guwahati : Ph: 0361-2738 811, 2735 640, guwahati@schandpublishing.com

m
Hyderabad : Ph: 040-4018 6018, hyderabad@schandpublishing.com

Li
Jaipur : Ph: 0141-2291 317, 2291 318, jaipur@schandpublishing.com
Jalandhar : Ph: 0181-4645 630, jalandhar@schandpublishing.com

ny
Kochi : Ph: 0484-2576 207, 2576 208, cochin@schandpublishing.com
Kolkata : Ph: 033-2335 7458, 2335 3914, kolkata@schandpublishing.com

pa
Lucknow : lucknow@schandpublishing.com
Mumbai : Ph: 022-2500 0297, mumbai@schandpublishing.com
Nagpur : Ph: 0712-2250 230, nagpur@schandpublishing.com
om
Patna : Ph: 0612-2260 011, patna@schandpublishing.com
Ranchi : ranchi@schandpublishing.com
C

Sahibabad : Ph: 0120-2771 238, info@schandpublishing.com


d

© S Chand And Company Limited, 1992


An

All rights reserved. No part of this publication may be reproduced or copied in any material form (including photocopying or storing it in any medium in form of graphics, electronic
or mechanical means and whether or not transient or incidental to some other use of this publication) without written permission of the publisher. Any breach of this will entail
legal action and prosecution without further notice.
d

Jurisdiction : All disputes with respect to this publication shall be subject to the jurisdiction of the Courts, Tribunals and Forums of New Delhi, India only.
n
ha

Fourteenth Edition (Old Syllabus) 1992


First Bifurcated Edition for Class IX 1993
Revised Edition 2016
C

Reprints 2016, 2017, 2018 (Twice), 2019 (Twice), 2020 (Twice)


S
©

ISBN : 978-93-52838-45-5 Product Code : SCS3IPH090MATAE20ICN

PRINTED IN INDIA
By Vikas Publishing House Private Limited, Plot 20/4, Site-IV, Industrial Area Sahibabad, Ghaziabad–201 010
and Published by S Chand And Company Limited, A-27, 2nd Floor, Mohan Co-operative Industrial Estate, New Delhi–110 044.
A Note on the Revised Edition
This edition is meant for the students of class IX and is written strictly according to the syllabus
prescribed by the Council for the Indian School Certificate Examinations, New Delhi.

Special Features of the Book:


1. The book takes full advantage of the clarity and consistency of modern terminology.
2. The development is logical, and the preparation of each new idea is based on the preceding material.
3. The clearly developed textual explanations are followed by appropriate solved examples.
4. The mathematical language is kept precise and correct.

d
5. Historical Notes have been interspersed throughout the text.

ite
6. The book reflects the authors' conviction that one learns mathematics by doing mathematics and

m
that a proper balance between theory and practice is essential for genuine understanding.

Li
7. The book uses S.I. Units.
8. Great pains have been taken to present the subject matter in a very easy to understand and easily

ny
comprehensible manner. To achieve this, we had sometimes to sacrifice brevity and give detailed
explanations to bring home to the students the finest aspects of every topic.
9. The treatment of ‘Statistics’ is very exhaustive and correct as it should be.
pa
om
ll methods have been copiously illustrated by the solved examples and all questions in the Exercises
10. All
have been carefully graded.
C

11. Summary of Key Facts given under the heading ‘Looking Back’ have been given to facilitate quick
revision and assimilation of the concepts learnt in the chapters.
d

12. Chapter Tests have been given for revision/self assessment at the end of every chapter/unit
An

containing Multiple Choice Questions (MCQ's)


(MCQ's).
d

Authors
n
ha
C
S
©

Disclaimer : While the authors of this book have made every effort to avoid any mistakes or omissions and have used their skill,
expertise and knowledge to the best of their capacity to provide accurate and updated information, the authors and S. Chand do not
give any representation or warranty with respect to the accuracy or completeness of the contents of this publication and are selling
this publication on the condition and understanding that they shall not be made liable in any manner whatsoever. S. Chand and the
authors expressly disclaim all and any liability/responsibility to any person, whether a purchaser or reader of this publication or not, in
respect of anything and everything forming part of the contents of this publication. S. Chand shall not be responsible for any errors,
omissions or damages arising out of the use of the information contained in this publication.
Further, the appearance of the personal name, location, place and incidence, if any; in the illustrations used herein is purely coincidental
and work of imagination. Thus the same should in no manner be termed as defamatory to any individual.
A Note to the Students
The following suggestions will help the students learn mathematics:
1. Do not try to read mathematics rapidly. In fact, force yourself to go slowly. A book on mathematics
should not be read in the same way as a novel or a history book.
2. Merely reading a book on mathematics is not sufficient. Mathematics can be learnt only by doing
the sums.
3. It may not be possible to grasp fully some of the ideas by reading just once. You may have to read
again and again to master completely some of the sections.
4. You must ask your teacher to help you if you fail to understand the subject-matter. Keep a pencil
and paper handy for making a note of your difficulties. Don't be afraid of asking your teacher for help.

d
A good teacher will be always too glad to help you.

ite
5. Try to understand definitions and principles. Try to restate them in your own words. Mathematical
ideas not understood properly cannot be used correctly.

m
6. If you fail to solve a sum, study the subject-matter again or the solved examples.

Li
7. Do your work neatly and systematically. Many mistakes in mathematics result from poor and
rough writing.

ny
8. There is nothing like rough work in mathematics. What you call rough work is in fact a part of the
working. All working must be shown while doing a sum.

pa
9. Mistakes pointed out in your work by your teacher are more important than the work itself. Before
doing the next home work, corrections in the previous home work must be done.
om
10. Lastly, do not lose heart too soon. Learning is a result of effort and perseverance.
C

Authors
d
An
n d
ha
C
S
©
The Syllabus
Aims:
1. To acquire knowledge and understanding of the terms, symbols, concepts, principles, processes, proofs, etc., of
mathematics.
2. To develop an understanding of mathematical concepts and their application to further studies in mathematics
and science.
3. To develop skills to apply mathematical knowledge to solve real life problems.
4. To develop the necessary skills to work with modern technological devices such as calculators and computers
in real life situations.
5. To develop drawing skills, skills of reading tables, charts and graphs.
6. To develop an interest in mathematics.

d
ite
CLASS IX
There will be one paper of two and a half hours duration carrying 80 marks and Internal Assessment of

m
20 marks.
The paper will be divided into two sections, Section I (40 marks) and Section II (40 marks).

Li
Section I: will consist of compulsory short answer questions.

ny
Section II: Candidates will be required to answer four out of seven questions.
The solution of a question may require the knowledge of more than one branch of the syllabus.

1. Pure Arithmetic
Rational and Irrational Numbers pa
om
Rational, irrational numbers as real numbers, their place in the number system. Surds and rationalization of surds.
Simplifying an expression by rationalizing the denominator. Representation of rational and irrational numbers on
C

the number line. Proof of irrationality of 2 , 3 and 55..


2. Commercial Mathematics
d

Compound Interest
An

(a) Compound interest as a repeated Simple Interest computation with a growing Principal. Use of
this in computing Amount over a period of 2 or 3 years.
n
 r 
d

(b) Use of Formula A = P 1 +  . Finding CI from the relation CI = A – P.


 100 
n

• Interest compounded half-yearly included.


ha

• Using the formula to find one quantity given different combinations of A, P, r, n, CI and SI; difference between
CI and SI type included.
C

• Rate of growth and depreciation.


Note: Paying back is equal installments, being given rate of interest and installment amount, not included.
S

3. Algebra
©

(i) Expansions
Recall of concepts learned in earlier classes.
(a ± b)2; (a ± b)3; (x ± a) (x ± b); (a ± b ± c)2
(ii) Factorisation
a2 – b2; a3 ± b3 ; ax2 + bx + c, by splitting the middle term.
(iii) Simultaneous Linear Equations in two variables. (With numerical coefficient only)
• Solving algebraically by:
– Elimination
– Substitution and
– Cross Multiplication method
• Solving simple problems by framing appropriate equations.
(iv) Indices/Exponents
Handling positive, fractional, negative and “zero” indices.
Simplification of expressions involving various exponents
am × an = am+n, am ÷ an = am–n, (am)n = amn, etc. Use of laws of exponents.
(v) Logarithms
(a) Logarithmic form vis-á-vis exponential form: interchanging.
(b) Laws of logarithms and their use.
Expansion of expression with the help of laws of logarithms
a 4 × b2
e.g., y=
c3
log y = 4 log a + 2 log b – 3 log c, etc.
4. Geometry
(i) Triangles
(a) Congruency: four cases SSS, SAS, AAS and RHS. Illustration through cutouts. Simple applications.

d
(b) Problems based on:

ite
• Angles opposite equal sides are equal and converse.
• If two sides of a triangle are unequal, then the greater angle is opposite the greater side and

m
converse.
• Sum of any two sides of a triangle is greater than the third side.

Li
• Of all straight lines that can be drawn to a given line from a point outside it, the perpendicular is the shortest.

ny
Proofs not required.
(c) Mid-point Theorem and its converse, equal intercept theorem

pa
(i) Proof and simple applications of mid-point theorem and its converse.
(ii) Equal intercept theorem: proof and simple application.
om
(d) Pythagoras Theorem
Area based proof and simple applications of Pythagoras Theorem and its converse.
C

(ii) Rectilinear Figures


(a) Proof and use of theorems on parallelogram.
d

• Both pairs of opposite sides equal (without proof).


An

• Both pairs of opposite angles equal.


• One pair of opposite sides equal and parallel (without proof).
d

• Diagonals bisect each other and bisect the parallelogram.


n

• Rhombus as a special parallelogram whose diagonals meet at right angles.


ha

• In a rectangle, diagonals are equal, in a square they are equal and meet at right angles.
(b) Construction of Polygons
C

Construction of quadrilaterals (including parallelograms and rhombus) and regular hexagon using ruler
and compasses only.
S

(c) Proof and use of Area theorems on parallelograms:


©

• Parallelograms on the same base and between the same parallels are equal in area.
• The area of a triangle is half that of a parallelogram on the same base and between the same parallels.
• Triangles between the same base and between the same parallels are equal in area (without proof).
• Triangles with equal areas on the same bases have equal corresponding altitudes.
(iii) Circle
(a) Chord properties
• A straight line drawn from the centre of a circle to bisect a chord which is not a diameter is at right
angles to the chord.
• The perpendicular to a chord from the centre bisects the chord (without proof).
• Equal chords are equidistant from the centre.
• Chords equidistant from the centre are equal (without proof).
• There is one and only one circle that passes through three given points not in a straight line.
(b) Arc and chord properties
• If two arcs subtend equal angles at the center, they are equal, and its converse.
• If two chords are equal, they cut off equal arcs, and its converse (without proof).
Note: Proofs of the theorems given above are to be taught unless specified otherwise.
5. Statistics
Introduction, collection of data, presentation of data, Graphical representation of data, Mean, Median of ungrouped data.
(i) Understanding and recognition of raw, arrayed and grouped data.
(ii) Tabulation of raw data using tally-marks.
(iii) Understanding and recognition of discrete and continuous variables.
(iv) Mean, median of ungrouped data.
(v) Class intervals, class boundaries and limits, frequency, frequency table, class size for grouped data.
(vi) Grouped frequency distributions: the need to and how to convert discontinuous intervals to continuous intervals.
(vii) Drawing a frequency polygon.

d
6. Mensuration

ite
Area and perimeter of a triangle and a quadrilateral. Area and circumference of a circle. Surface area and volume of
Cube and Cuboids.

m
(a) Area and perimeter of triangle (including Heron’s formula), rhombus, parallelogram and trapezium.
(b) Circle: Area and Circumference. Direct application problems including Inner an Outer area.

Li
Areas of sectors of circles other than quarter circle and semicircle are not included.
(c) Surface area and volume of 3D solids: cube and cuboid including problems of type involving:

ny
• Different internal and external dimensions of the solid.
• Cost.

pa
• Concept of volume being equal to area of cross-section × height.
• Open/closed cubes/cuboids.
7. Trigonometry
om
(a) Trigonometric Ratios : sine, cosine, tangent of an angle and their reciprocals.
(b) Trigonometric ratios of standard angles – 0, 30, 45, 60, 90 degrees. Evaluation of an expression involving these ratios.
C

(c) Simple 2D problems involving one right-angled triangle.


(d) Concept of trigonometric ratios of complementary angles and their direct application:
d

sin A = cos (90 – A), cos A = sin (90 – A)


An

tan A = cot (90 – A), cot A = tan (90 – A)


A)
sec A = cosec (90 – A), cosec A = sec (90 – A
A))
8. Coordinate Geometry
d

Cartesian system, Plotting a points in the plane for given coordinates, solving simultaneous linear equations in 2
variables graphically and finding the distance between two points using distance formula.
n

(a) Dependent and independent variables.


ha

(b) Ordered pairs, coordinates of points and plotting them in the Cartesian plane.
(c) Solutions of Simultaneous Linear Equations graphically.
C

(d) Distance formula.


S

Internal Assessment
©

A minimum of two assignments are to be done during the year as prescribed by the teacher.
Suggested Assignments
• Conduct a survey of a group of students and represent it graphically - height, weight, number of family members,
pocket money, etc.
• Planning delivery routes for a postman/milkman.
• Running a tuck shop/canteen.
• Study ways of raising a loan to buy a car or hourse, e.g., bank loan or purchase a refrigerator or a television set
through hire purchase.
• Cutting a circle into equal sections of a small central angle to find the area of a circle by using the formula A = πr2.
• To use flat cutouts to form cube, cuboids and pyramids to obtain formulae for volume and total surface area.
• Draw a circle of radius r on a ½ cm graph paper, and then on a 2 mm graph paper. Estimate the area enclosed
in each case by actually counting the squares. Now try out with circles of different radii. Establish the pattern,
if any, between the two observed values and the theoretical value (area = πr2). Any modifications?
SI Units, Symbols and Abbreviations
1. (a) Units may be written in full or using the agreed symbols, but no other abbreviations may be used.
(b) The letter ‘s’ is never added to symbols to indicate the plural form.
(c) A full stop is not written after symbols for units unless it occurs at the end of a sentence.
(d) When unit symbols are combined as a quotient, e.g., metre per second, it is recommended that they
be written as m/s or better still as m s–1.
(e) Three decimal signs are in common international use, the full point, the mid-point and the comma.
Since the full point is sometimes used for multiplication and the comma for spacing digits in large
numbers, we shall be on the safe side if we use the mid-point for decimals.
2. Names and symbols

d
(a) In general

ite
implies that ⇒ logically equivalent ⇔
identically equal to ≡ to approximately equal to = or ≈ or >>

m
(b) In set language

Li
belongs to ∈ does not belong to ∉
is equivalent to ↔ is not equivalent to ↔

ny
union ∪ intersection ∩

pa
contains ⊃ is contained in ⊂
universal set ξ the empty set φ
om
natural (counting) numbers N whole numbers W
integers I or Z real numbers R
C

rational numbers Q irrational numbers Q


(c) In measures
d

kilometre km metre m
An

centimetre cm millimetre mm
kilogram kg gram g
d

litre l centilitre cl
n

2
square kilometre km square metre m2
ha

square centimetre cm2 hectare ha


3
cubic metre m cubic centimetre cm3
C

kilometres per hour km/h or km h–1 metre per second m/s or m s–1
S

SI Units
©

All measurements in Physics and Mathematics are related to three chosen fundamental quantities
of ‘length, mass and time’. Until recently, workers in various countries used different systems of units.
Fortunately, this unsatisfactory situation has now been changed by the efforts of various international
committees of scientists who have met for discussion regularly over many years.
In 1960, the Conference Generale des Poids et Measures (the General Conference of Weights and
Measures) recommended that everyone should use a metric system of measurement called the International
System of Units (abbreviated SI in all languages). The SI units are derived from the earlier MKS-system, so
called because its first-three basic units are the metre (m), the kilogramme (kg) and the second (s).
For the purpose of this book the following points are sufficient to remember:
(a) The letter ‘s’ is never added to the symbols for units to indicate a plural form. Thus, one writes 5 m
and not 5 ms for 5 metres or 3.7 cm and not 3.7 cms for 3.7 centimetres, etc.
(b) A full stop is not written after symbols for units because these are not abbreviations except when it is
occurring in the ordinary way when they are at the end of a sentence. For example, cm and not cm.,
kg and not kg., are to be written.
(c) The dot notation will not normally be used to indicate multiplication. Exceptions will include such
cases as in Geometry the area of a lettered rectangle, e.g., LM.MN, etc.
2
(d) The unit symbol for area in metres will be written as m and not sq. metre. Likewise the unit symbol for
volume in metres, centimetres, etc., will be written as, m3, cm3 etc., and not cu. m., cu. cm.
(e) Miles, feet, inches, etc., will only be used to a very limited extent according to the subject, these f.p.s.
units will effectively disappear from question papers in time.
–1
(f) In everyday examples, e.g., car speeds, km/h or km h will be used as appropriate. Minutes (min.),
years (yr.) will also be given for precise scientific usage, the ‘second’ will be preferred.
 1 '  1 "
(g) The degree (°) will continue to be in use, also will minutes:   and seconds   .

d
 60   60 

ite
The following are the S.I. and other metric units commonly used in Mathematics:
Length Area

m
Unit Abbreviation Unit Abbreviation

Li
millimetre .................. mm square centimetre ................ cm2
centimetre .................. cm square metre ................ m2

ny
metre .................. m square kilometre ................ km2
kilometre .................. km
Volume and Capacity
cubic centimetre.......... cm
3
Weight
milligramme pa ................ mg
om
3
cubic metre.................. m gramme ................ g
decilitre .................. dl kilogramme ................ kg
C

3
litre .................. l (1000 cm ) tonne (1000 kg) ................ t
kilolitre .................. kl (1000 l )
d

Time Speed
An

second .................. s centimetre/second ................ cm/s or cm s–1


minute .................. min centimetre/minute ................ cm/min or cm min–1
d

hour .................. h centimetre/hour ................ cm/h or cm h–1


n

day .................. dy metre/second ................ m/s or m s–1


ha

week .................. wk metre/minute ................ m/min or m min–1


year .................. yr metre/hour ................ m/h or m h–1
C

3:15 a.m .................. 03 15 kilometre/second ................ km/s or km s–1


3:15 p.m .................. 15 15 kilometre/minute ................ km/min or km min–1
S

kilometre/hour ................ km/h or km h–1


©

Land areas
In the metric system, land areas are measured in hectare (ha). An hectare is 1 square hectometre or
ten-thousand square metres. Thus,
1 ha = 1 hm2 = 10000 m2
1 are = 100 m2, 1 centare = 1 m2
Contents
UNIT 1 : PURE ARITHMETIC
Chapter 1. Rational and Irrational Numbers 1.1 – 1.21

UNIT 2 : COMMERCIAL MATHEMATICS


Chapter 2. Compound Interest 2.1 – 2.14

UNIT 3 : ALGEBRA

d
Chapter 3. Expansions 3.1 – 3.10

ite
Chapter 4. Factorisation 4.1 – 4.13
Chapter 5. Simultaneous Linear Equations in two Variables 5.1 – 5.17

m
Chapter 6. Indices/Exponents 6.1 – 6.11

Li
Chapter 7. Logarithms 7.1 – 7.9

ny
UNIT 4 : GEOMETRY

pa
Chapter 8. Triangles 8.1 – 8.22
Chapter 9. Mid-Point and Intercept Theorems 9.1 – 9.9
om
Chapter 10. Pythagoras Theorem 10.1 – 10.11
Chapter 11. Rectilinear Figures 11.1 – 11.24
C

Chapter 12. Area Theorems 12.1 – 12.16


Chapter 13. Circle 13.1 – 13.18
d
An

UNIT 5 : STATISTICS
Chapter 14. Introduction, Data and Frequency Distributions 14.1 – 14.9
d

Chapter 15. Mean, Median and Frequency Polygon 15.1 – 15.15


n
ha

UNIT 6 : MENSURATION
C

Chapter 16. Area of Plane Figures 16.1 – 16.16


Chapter 17. Circle – Circumference and Area 17.1 – 17.12
S

Chapter 18. Surface Area and Volume of 3D Solids 18.1 – 18.12


©

UNIT 7 : TRIGONOMETRY
Chapter 19. Trigonometrical Ratios 19.1 – 19.20

UNIT 8 : COORdINATE GEOMETRY


Chapter 20. Coordinates and Solution of Simultaneous Linear Equations Graphically and 20.1 – 20.14
Distance Formula
UNIT 1 PURE ARITHMETIC

RATIONAL AND
1
CHAPTER IRRATIONAL NUMBERS

d
ite
RATIONAL NUMBERS

m
NEED FOR RATIONAL NUMBERS
As civilisation developed, there was a need to divide quantities into parts. After having extended the system of natural

Li
numbers so as to include zero and negative integers, man soon discovered that he could not do all of his arithmetic
with just the integers. The system of integers suffered from the defect that division is not always possible within the

ny
system. For example, to problems such as 3 ÷ 5 or −77 ÷ 3, he had no answer. That is to say, no integer could be found
to fill in the blank : 5 × ... = 3 or 3 × .... = −7.
7. Therefore man felt the need to go beyond integers and construct a new

pa
system which included integers and in which all divisions could be carried out. The numbers that were created were
called rational numbers. Rational numbers include all positive integers, positive fractions, zero, negative integers
om
and negative fractions. The letter Q is used to name the set of rational numbers. The word ‘rational’ is derived from
the word ‘ratio’.
C

a
Definition. A rational number is any number that can be named in the form where a and b are integers and
b
d

b ≠ 0.
An

3 −5  7  −3 
Thus, , and 7  =  , − 3  =  are all rational numbers.
7 8  1  1 
4 (= 2), 9, 16, 25 etc. are all
d

Note. The square root of every perfect square number is rational. Thus,
n

rational.
ha

Properties of operations on rational numbers


1. It is easy to verify that the set Q of rational numbers is closed for addition, subtraction, multiplication and
C

division, except when the divisor is zero.


3 5 41
S

Thus, (i)(i)) + =
(i , which is a rational number.
4 7 28
©

3 5 1
ii)) − =
(ii)
ii , which is a rational number.
4 7 28
3 5 15
(iii) × = , which is a rational number.
4 7 28
3 5 3 7 21
(iv) ÷ = × = , which is a rational number.
5 7 5 5 25
Thus, when a rational number is divided by any non-zero rational, the result is always a rational number.
2. The set of rational numbers is commutative for the operations of addition and multiplication,
1 3 3 1 22 1 3 3 1 3
Thus, + = + = ; × = × =
5 7 7 5 35 5 7 7 5 35
Ch 1–2 ICSE MATHEMATICS–IX

3. The set of rational numbers is associative for the operations of addition and multiplication.
1 1 1 1 1 1 1 1 1 1 1 1
Thus,  +  + = +  +  ;  ×  × = ×  × 
 2 3 4 2 3 4  2 3 4 2 3 4
4. Identity elements 0 and 1 exist for the operation of addition and multiplication respectively.
5. (i) Additive inverse. For every rational number a there exists a rational number −a, such that a + (− a) = 0.
5  −5  −5 5
Thus, +   = 0, so is the additive inverse of .
8  8  8 8
1
(ii) Multiplicative inverse. For every non-zero rational number a there exists a rational number such that
a

d
1 3 8 8 3
a.= 1. Thus, × = 1, so is the multiplicative inverse of .

ite
a 8 3 3 8
6. The property of distributivity holds in the set of rational numbers.

m
2 1 1 2 1 2 1

Li
Thus, × +  =  ×  +  × .
7 6 9 7 6 7 9

ny
Denseness property of rational numbers
Rational numbers are different from integers in many ways. For example, on a number line there is not always
another integer between any two given integers.
pa
om

There are exactly four integers between 0 and 5, viz.


viz.,, 1, 2, 3 and 4. There are no integers between 0 and 1.
C

Between any two rational numbers, however, it is always possible to find another rational number. For example
d

1 3 1 3
consider and . You can find the rational number that is one half of the way from and by adding one half
An

2 4 2 4
1 3 1
the difference between and to .
2 4 2
n d

1 1 3 1 1 1 1 1 1 5
+  − = +  = + =
ha

2 2 4 2 2 2 4 2 8 8
C

1 5 3
2 8 4
S

Fig. 1.02 1 5
You can then use this method to find the number that is one half of the way from to .
2 8
©

1 1 5 1 1 1 1 1 1 9
+  − = +  = + =
2 2  8 2  2 2  8  2 16 16

1 9 5 3
2 16 8 4
Fig. 1.03
17 1 9 33 1 17
Similarly, is one half of the way from to , is one half of the way from to , and so on.
32 2 16 64 2 32

1 33 17 9 5 3
2 64 32 16 8 4
Fig. 1.04
rATIonAl And IrrATIonAl nuMbErS Ch 1–3

1 3
Therefore, there exist at least four rational numbers between and . In fact, continuing with this process, you
2 4
1 3
could find an infinite number of rational numbers between and .
2 4
In general, given two rational numbers a and b, a < b, the number that is one half of the way from a to b is
1
a+ (b − a).
2
1
The number one third of the way from a to b is a + (b − a) ; and so on. These formulas suggest the density
3
property of rational numbers, which is stated below:

d
ite
Denseness Property of Rational Numbers:
Between any two different rational numbers, there are infinitely many rational numbers.

m
Illustrative examples

Li
1 7
Ex. 1. Find a rational number between and .

ny
5 10
1 7
Sol. You may choose any number, one-half, one-third, one-fourth, one-fifth etc. of the way from to . Suppose
5 10

pa
1 7
you choose the number one fourth of the way from to .
5 10
om
1 1  7 1 1 1 1 1 1 1 3
+  − = +  = + =
5 4  10 5  5 4  2  5 8 40
C

1 7 13
∴ One rational number between to is .
5 10 40
d
An

 To find many rational numbers between two given distinct rational numbers.
Method :
1 1 1
d

Let the given rational numbers be a and b. Then q1 = (a + b), q2 = (q1 + b), q3 = (q2 + b),
2 2 2
n

1
ha

q4 = (q + b),
), and so on.
2 3
C

In this manner we can find as many rational numbers as we please between two given distinct rational numbers.
S

1 1
Ex. 2. Give three rational numbers lying between and .
3 2
©

1 1 1 1 1 1 5 5 1 5 1
Sol. Rational numbers between and =  + =  =  Hence, < < .
3 2 2 3 2  2  6  12 3 12 2

1 5 1 1 5  1  4 + 5 9 1 9 5
Rational numbers between and =  +=   =  . Hence < < .
3 12 2  3 12  2  12  24 3 24 12

5 1 1  5 1  1  5 + 6  11 5 11 1
Rational numbers between and =  = +   =  . Hence < < .
12 2 2  12 2  2  12  24 12 24 2

1 1 9 5 11
Thus, three rational numbers between and are , , .
3 2 24 12 24
Ch 1–4 ICSE MATHEMATICS–IX

Short cut :
a c a+c
Note. If and are two distinct rational numbers, then is also a rational number lying between
b d b+d
a c
and . Therefore, a rational number lying between two distinct given rational numbers is easily obtained
b d
on adding the numerators and denominators. Thus, in the above example,
1 1 1+1 2
Rational number lying between and = =
3 2 3+ 2 5
1 2 1+ 2 3
Rational number lying between and = =
3 5 3+5 8

d
2 1 2 +1 3
Rational number lying between and = =

ite
5 2 5+2 7
1 1 2 3 3
∴ Three rational numbers lying between and are , and .

m
3 2 5 8 7

Li
1 1
Ex. 3. Find four rational numbers between and .
4 3

ny
Sol. Let q1, q2, q3 and q4 be the four required rational numbers.

Then, q1 =
11 1 1 3+ 4 7
 =+   = 
2  4 3  2  12  24

1 7 1
< < .
4 24 3 pa
om
1 1 7  1 6+7 1 3 1 13 7
q2 =  + =   =  ∴ < < .
2  4 24  2  24  48 4 48 24
C

1  7 1  1  7 + 8  15 7 15 1
q3 =  = +   =  ∴ < < .
2  24 3  2  24  48 24 48 3
d
An

1  15 1  1  15 + 16  31 15 31 1
q4 =  = +   =  ∴ < < .
2  48 3  2  48  96 48 96 3
d

1 1 13 7 15 31
Thus, four rational numbers between and are , , , .
n

4 3 48 24 48 96
ha

Another method :
The following is a better method :
C

Suppose you have to find n rational numbers between any two rational numbers a and b, when a < b.
S

Step 1. Find the gap between a and b. This is equal to (b – a).


Step 2. Divide the gap into (n + 1) smaller equal gaps by dividing (b – a) by (n + 1).
©

b−a a b
Thus, you have .
n +1
Step 3. Multiply the gap size by 1, 2, 3,... until n, and add a to each product to get n new rational numbers.
b−a b−a b−a b−a
Thus, n new rational numbers are a + 1. , a + 2. , a + 3. , ..., a + n. ,
n +1 n +1 n +1 n +1
1 1
Ex. 4. Find 7 rational numbers between and .
3 2
1 1 3−2 1
Sol. Step 1. Gap = − = = a b
2 3 6 6 a + 1.
b–a
a + 3.
b–a ..... a + n . b – a
n+1 n+1 n+1
b–a b–a
a + 2. a + 4.
n+1 n+1
rATIonAl And IrrATIonAl nuMbErS Ch 1–5

1 1 1 1
Step 2. Dividing the gap by (7 + 1), i.e., 8 we get ÷8= × =
6 6 8 48
1 1 1 1 1 1
Step 3. Then the seven rational numbers between and are + 1 × , + 2 × ,
3 2 3 48 3 48
1 1 1 1 1 1 1 1 1 1 17 18 19 20 21 22 23
+ 3 × , + 4 × , + 5 × , + 6 × , + 7 × , i.e., , , , , , , .
3 48 3 48 3 48 3 48 3 48 48 48 48 48 48 48 48
−2 −1
Ex. 5. Find three rational numbers lying between and .
5 5
Sol. Let q1, q2, and q3 be the three required rational numbers. Then
1  −2 1  1  −2 − 1  1  −3  −3 −2 −3 −1 1

d
q1 =  − =  =  = ∴ < < Using ((aa + b)
b)
2  5 5  2  5  2  5  10 5 10 5 2

ite
1  −2 3  1  −4 − 3  −7 −2 −7 −3
q2 =  − =  = ∴ < <

m
2  5 10  2  10  20 5 20 10

Li
1  −3 1  1  −3 − 2  −5 −3 −5 −1
q3 =  − =  = ∴ < <
2  10 5  2  10  20 20 20 5

ny
−2 −1 −7 −3 −5
Hence, three rational numbers between and are , , .
5 5 20 10 20
Third Method : Common Denominator Method
pa
om
–2 1
Ex. 6. Find 10 rational numbers between and .
5 2
Method :
C

Convert the denominators of the given rational numbers to a common denominator so that the difference of their
numerators may be more that the number of rational numbers to be found out.
d

Sol. Here, we have to find 10 rational numbers. So, change the denominators 5 and 2 to a common denominator
An

so that the difference of numerators –2 and 1 may be more than 10.


–2 –8 1 10
We have, = and =
d

5 20 2 20
n

Difference of 10 and –8 is 18, which is > 10 and common denominator = 20


ha

–8 10 –7 –6 –5 –4 –3 –2 –1 1 2
10 rational numbers between and are , , , , , , , 0, , .
20 20 20 20 20 20 20 20 20 20 20
C

Decimal representation of rational numbers


a
Because all rational numbers can be written in the form , a rational number can be expressed as a decimal by
S

b
simply dividing a by b.b This is illustrated in the following examples.
©

5 7
=5÷8 = 7 ÷ 11
8 11
0.625 0.6363..
8 ) 5.000 11 ) 7.0000..
–48 –66
20 40
– 16 – 33
40 70
– 40 5 – 66
∴ = 0.625
0 8 40
– 33 7
∴ = 0.6363...
7 11
Ch 1–6 ICSE MATHEMATICS–IX

In the division of 5 by 8, the quotient 0.625 is called a terminating decimal because the divison process terminates
when a final remainder of 0 is reached.
In the divsion of 7 by 11, on the other hand, the quotient 0.6363 ... is a non-terminating decimal. The division
process never terminates because the remainders 4 and 7 (printed in bold) keep appearing, and a remainder of 0 is
never reached. The quotient is called a repeating decimal because it is a non-terminating decimal in which the same
block of digits repeats without end.
23 2
If you divide 23 by 16, you will get = 1.4375 and if you divide 2 by 13 you will get = 0.153846153846 ...
16 13
7 2
The decimal for is 0·6363... It repeats a block of two digits. The decimal for is 0.153846153846... It repeats
11 13
a block of six digits. To show this, a bar is used to indicate the block of digits thatt repeats. Therefore,
7 2

d
= 0.6363 .... = 0.63 , = 0.153846153846 ... = 0.153846

ite
11 13
..
Another notation for a repeating decimal is putting a dot at the top of the end digits, like 0.6363 ... = 0. 63 , and

m
. .
0.153846153846... = 0.15384 6 .

Li
Every rational number can be named by either a terminating decimal or a repeating decimal.

ny
All terminating and repeating decimals are called rational numbers. Decimals that do not terminate and are
non-repeating are called irrational numbers. The prefix “ir” means “not”, therefore, irrational numbers are not

pa
rationals; they include all numbers that cannot be written as fractions. A simple example of an irrational number is
0·101001000100001... Here, because of the increasing number of 0’s between the digits of 1, there is no group of
om
digits that repeats.

Changing decimals into fractions


C

Case I. When the decimals are terminating


d

You can write a terminating decimal as a fraction with a denominator that is a power of 10.
An

For example,
5 1
0.5 = = ← Rewrite the fraction in lowest terms.
10 2
d

47 4 2 1010 101
n

0.47 = , 3.4 = 3 = 3 , 10.10 = = .


100 10 5 100 10
ha

Case II. When the decimals are repeating


C

Type 1. Decimals in which all the digits after the decimal point are repeated.
S

Ex. 7. Express each decimal as a fraction in simplest form.


©

(i) 0.8 (ii) 1.27 (iii) 0.407


Sol.
(i) Let n = 0.8 = 0.888 ... (i)
10n = 10 × 0.888... = 8.888... = 8.8 ... (ii)
Subtracting (i) from (ii), we get
9n = 8
Think
8 Since there is one digit (8) in the repeated block we
⇒ n =
9 multiply both members, of n = 0.8 , by 10. This shifts
i.e., 0.8 = 8 the repeating digit one place to the left. The subtraction
9 produces a terminating decimal numeral.
rATIonAl And IrrATIonAl nuMbErS Ch 1–7

(ii) Let n = 1.27 = 1.2727... ... (i)


Since there are two digits (27) in the repeated block,
Multiply : 100n = 127.27 ... (ii) we multiply both members, of n = 1.27, by 102, or 100.
This shifts the repeating block of digits two places to the
Subtracting (i) from (ii), we get, 99n = 126 left. The subtraction produces a terminating decimal
126 14 numeral.
Divide : n = = .
99 11
(iii) Let n = 0 ⋅ 407 ... (i)
Since the block of repeating digits contains 3 digits, we
Multiply : 1000n = 407.407 ... (ii) 3
multiply by 10 , or 1000.
Subtracting (i) from (ii), we get, 999n = 407

d
407 11
∴ n = = .

ite
999 27

m
Rule. Write the repeated digit only once in the numerator and write as many nines in the denominator as is
the number of repeated digits.

Li
Applying this rule to the above examples, we get

ny
8 27 3 14 407 11
0.8 = ; 1.27 =+
1 =+1 = ; 0.407 = = .
9 99 11 11 999 27

pa
Type 2. Decimals in which at least one digit after the decimal point is not repeated.
om
Ex. 8. Convert (i) 0.47 , (ii) 4.2348 into fractions in the simplest form.
Method :
C

Step 1. Let the given recurring decimal = x


Step 2. Count the number of digits after the decimal point while do not have bar over them. Suppose there are
d

n such digits.
An

n
Step 3. Multiply both sides by 10 . Then only the repeating decimal remains on the right side.
Step 4. Now, apply the same method as in case of Type I (Pure repeating decimal) and obtain the value of x.
d

Sol. (i) Step 1. Let x = 0.47 = 0.47777... ... (i)


n

Step 2. 10x = 4.777..... ... (ii)


ha

Thinking Process
Step 3. In (ii),
), above, only one digit, (i.e., 7) is repeating
There is only one non-repeating digit, viz., 4
on RHS, so we multiply both sides of (ii)( by 10.
C

after the decimal point, so we multiply both


sides by 101, i.e., 10.
∴ 10xx = 47.777.....
10 ...(iii)
S

ii)) from ((iii), we get


Subtracting (ii
((ii)
43
90x = 43 ⇒ x =
©

43 90
Hence, 0.47 = .
90
(ii) Let x = 4.2348 ... (i) Think
There are 2 digits on the right side of the decimal
100x = 100 × 4.23484848... point which are without bar, so we multiply both
⇒ x = 423.484848... ...(ii) sides by 102, i.e., 100.
Since there are 2 digits in the repeated block after the decimal point,
so multiplying both sides of (ii) by 100, we get
10000x = 42348.4848.. ...(iii)
Subtracting (ii) from (iii), we get
41925 559
9900x = 41925 ⇒ x= =
9900 132
Ch 1–8 ICSE MATHEMATICS–IX

Quick method to get the answer


Consider the decimal part. Subtract the non-repeating part from the whole and divide by as many nines as are
the repeating digits × as many tens as are non-repeating digits.
327 – 32 295 59 959
Thus, 5.327 = 5 + =5+ =5+ =
900 900 180 900
4868 – 48 4820 241
0.4868 = = =
9900 9900 495
1928 – 1 1925 385 10375
5.1926 = 5 + =5+ =5+ =
9990 9990 1998 1998
p
Ex. 9. Express the following in the form of , where p and q are integers and q ≠ 0.
q

d
(i) 0.134 (ii) 0.00323232... (NCERT Exemplar)

ite
Sol. (i) Let x = 0.134 Think
⇒ x = 0.1343434... ... (i) 1 digit after the decimal point over which there

m
is no bar, so we multiply by 10.
⇒ 10x = 1.343434... ... (ii)

Li
2 digits in the repeating block after the decimal
⇒ 1000x = 134.3434... ...(iii) point, so multiply by 100.

ny
Subtracting (ii) from (iii), we get
133
990x = 133 ⇒ x=

pa
990 Think
(ii) Let x = 0.00323232... (i))
... (i)
(i 2 digits after the decimal point without bar.
om
⇒ 100x = 100 × 0.00323232...
⇒ 100x = 0.3232... ii
...(ii)
...(ii
...(
C

⇒ 100 × 100x = 100 × 0.3232... 2 digits after the decimal point repeating block,
⇒ 10000x = 32.3232... ...(iii) so multiply both sides by 100.
d

On subtracting (ii) from (iii), we get


An

32 8
9900x = 32 ⇒ x = =
9900 2475
d

Ex. 10. Find the value of 2.6 − 0.9 .


n

Sol. Let x = 2.6 = 2.66 ... (i) Let y = 0.9 = 0.99 ... (i)
ha

10x = 26.6 ... (ii) 10y = 9.9 ... (ii)


C

Subtracting (i)) from ((ii),


), we get Subtracting (i) from (ii), we get
9x = 24
9x 9y = 9
S

24 8 9
∴ x = = ∴ y = =1
©

9 3 9
8 8−3 5
∴ 2.6 − 0.9 = x − y = − 1 = = .
3 3 3
p
Ex. 11. Explain 0.6 + 0.7 + 0.47 in the form of , where p and q are integers and q ≠ 0. (NCERT Exemplar)
q
6 3
Sol. 0.6 = =
10 5
To find 0.7 , let x = 0.7 = 0.777 ... (i)
⇒ 10x = 7.777... ... (ii) (Multiplying both sides by 10)
Subtracting (i) from (ii), we get
7
⇒ 9x = 7 ⇒ x=
9
rATIonAl And IrrATIonAl nuMbErS Ch 1–9

To find 0.47 , let y = 0.47 = 0.4777... ... (i)


⇒ 10y =4.777... ... (ii) (Multiplying both sides by 10)
⇒ 100y =47.777... ... (iii) (Multiplying both sides by 10)
Subtracting (ii) from (iii),
we get
⇒ 90y =43
43
⇒ y =
90
3 7 43
∴ 0.6 + 0.7 + 0.47 = + +
5 9 90
54 + 70 + 43 167
= =

d
90 90

ite
ExErCIsE 1 (a)

m
1 3 12

Li
1. (i) Find a rational number between and . 7. Express as decimal fraction.
2 4 125
.
8. Find a vulgar fraction equivalent to 0·03 .

ny
(ii) Find two rational numbers between 0·1 and
0.2. 9. Express the following rational numbers in the

pa
(iii) How many rational numbers can you find p
form , p, q are integers, q ≠ 0.
between two given rational numbers? q
2. Find two rational numbers between
om
4 7 3 1 ((ii 6.46
(i) (ii) 0.136 (iii) 3.146 (iv) −5.12
(i) and , (ii) and 1 .
5 13 4 5
10. Write the terminating decimal numeral for the
C

3. Find three rational numbers between 0 and 0.2. given rational numbers.
4. Find three rational numbers between 3 and 4. 7 29 17
d

5. Find the rational number that is one seventh of (i) (ii) (iii)
An

4 50 32
3 3
the way from 1 to 4 . 11. Write the repeating decimal for each of the
4 8
1 following, and use a bar to show the repetend.
d

6. Find four rational numbers between −1 and − . 1 −4 1


2 (i) (ii) (iii)
n

9 3 6
ha

ANsWErs
C

5 3 7 87 191 39 87
1. (i) ((ii) , (Answers may vary) (iii) infinite 2. (i) , (ii) ,
8 20 40 130 260 40 80
S

13 7 15 1 −3 −7 −5 −9
3. 0.1, 0.05, 0.15 4. , , 5. 2 6. , , , 7. 0.096
©

4 2 4 8 4 8 8 16
1 640 3 3143 169
8. 9. (i) (ii) (iii) (iv) −
30 99 22 999 33
10. (i) 1.75 (ii) 0.58 (iii) 0.53125
11. (i) 0.1 (ii) −1.3 (iii) 0.16

IrrATIONAL NUMBErs
• Real numbers that are not rational are called irrational numbers. Unlike rational numbers, they cannot be
a
represented in the form where a and b are integers and b ≠ 0.
b
Ch 1–10 ICSE MATHEMATICS–IX

• Irrational numbers as non-repeating, non-terminating decimals


You have learnt that one of the chief properties of the system of rational numbers is that every such number can
be represented as a repeating or a terminating decimal. Certainly, it is possible to construct a decimal numeral that is
non-repeating and non-terminating, i.e., which is a non-repeating infinite decimal.
(i) Start with 0·43
(ii) Annex the digits 433 making 0·43433
(iii) Annex the digits 4333 making 0·434334333
(iv) Continue annexing digits following this pattern :
0·43433433343333
If we continue the process indefinitely, we will construct a decimal numeral having an endless succession of
different blocks of digits. The number so constructed is not a rational number because a rational number can always

d
be expressed as a repeating or a terminating decimal. We call such non-repeating decimals irrational numbers
numbers. Notice

ite
that the word “irrational” is formed by attaching the prefix ir-, which means “not” to the word “rational”. Thus, an
irrational number is not a rational number. The word ‘rational’ is derived from the word ‘ratio’
‘ratio’.

m
Li
Irrational numbers are represented by non-terminating non-repeating decimals.
From the example of the irrational number taken above, you should not erroneously conclude that there is always

ny
some set pattern in the sequence of digits in the non-repeating infinite decimals. To fortify our point, we will now tell
you about irrational numbers whose decimal numerals do not take any systematic form. As you will see, square roots
such as 2 , 3 , 5, are such numbers.
pa
om
(i) Draw a number line l as shown. m

(ii) Mark the point at 0 as A and on l take AB = 1 unit.


(iii) At B, draw a ray m perpendicular to l.
C

(iv) On m draw a line segment BC,, one unit long.


d

C
(v) Draw segment AC.
An

(vi) With A as centre and radius AC,, draw circular arc which intersects l. A
B
1
D
l
D.
Call the point of intersection D. 0 1 1 2 3

Now two questions arise :


n d

(1) To what number (if any) does the point D correspond?


ha

(2) Is this number a rational number ?


Consider the first question, “To what number does the point D correspond ?”
C

AB = BC = 1
2 2 2
(AC = (AB) + (BC)
(AC
(AC) Pythagoras’ Theorem
S

2 2 2
AC = 1 + 1
©

AC2 = 2
AC = 2 , so AD = 2
Therefore, the point D corresponds to the number 2.
If you find the square root of 2 by the division method, you will find that 2 can be represented by the non-
repeating decimal 1.4142143... and so it is irrational.
Similarly, 3 = 1.732050... is an irrational number.
17 = 4.12310562561766... is an irrational number.
119 = 10.908712114635... is an irrational number.
rATIonAl And IrrATIonAl nuMbErS Ch 1–11

Irrational numbers occur as the square roots of certain numbers. They also result from many other mathematical
processes. For example, the ratio of the circumference of any circle to its diameter is the irrational number π.
π = 3.14159 2653589 7932 384626433832 7950 2884 19716 939 9375 1058 2097494...

The wheel of Theodorus


Theodorus of Cyrene, who lived around 425 B.C., was a philospher of ancient Greece. It is said that he discovered
the construction below, which is therefore called the ‘the wheel of Theodorus.’
1
1
1

d
4=2
1

ite
5 3

m
2 1

Li
1 6

1=1

ny
Fig. 1.09

Placement of 2, 3, 5 on the number line

On the number line we construct 2, 3, 5 as follows: pa


om
C
d
An
n d

On the number line X ′OX


OX let O represent zero, L represents 1 unit, M represents 2 units, and N represent 3 units.
ha

Thus, OL = 1, OM = 2, ON = 3.
Let OY be perpendicular to OX at O . Let OT = 1 unit. LA is perpendicular to OL, and LA = 1 unit. Join OA.
C

By Pythagoras Theorem, OA = OL2 + LA2 = 1+1 = 2 with O as centre and OA as radius draw an arc cutting
S

the number line OX at B


B.. Thus, OB = OA = 2.
©

At B draw the perpendicular BC equal to 1 unit. Join OC.

( 2)
2
By Pythagoras Theorem, OC = OB 2 + BC 2 = + 12 = 2 + 1= 3 . With O as centre and OC as radius

draw an arc cutting the number line OX at D. Thus, OD = OC = 3.


At M draw the perpendicular ME equal to 1 unit.

Join OE. By Pythagoras Theorem, OE = OM 2 + ME 2 = 22 + 12 = 4 + 1= 5.

With O as centre and OE as radius draw an arc cutting the number line OX at F. Then OF
= OE
= 5.

=
Hence, OB 2, OD
= 3, OF
= 5.
Ch 1–12 ICSE MATHEMATICS–IX

ThE rEAL NUMBErs


If we combine the rational numbers and the irrational numbers, we get real numbers which we denote by R.
The following diagram shows the relationships among the different kinds of numbers of the real number system.
Real Numbers

↓ ↓
Rational Numbers Irrational Numbers

↓ ↓
Integers Non-integers
↓ Rationals

d
↓ ↓ ↓

ite
Zero Positive Negative

m
Integers Integers

Li
Non-Zero
Whole Numbers

ny
Or
(Natural Numbers)

pa
om
REAl NuMbERS
bERS
b ERS (R)
Rational Numbers (Q) Irrational Numbers
C

0.2 5 11
3 −7 2
d

11
An

0
(N) 5
3 3
d

7.213 3 7 log10 3
n
ha

In the diagram, note the following relationships among the sets of numbers.
1. A real number ((R)
R)) is either a rational number or an irrational number. Thus, the real number 2 is a rational
R
C

number while the real number 2 is irrational. 3


S

2. A rational number ((Q) is either an integer or a non-integer. Thus, the rational number is an integer while
3 1
the rational number is a non-integer.
©

4
3. An integer (Z ) is either positive, negative, or zero. Thus, the integer 3 is positive, the integer −3 is negative,
and the integer 0, is neither positive nor negative.
Z = ..., –3, –2, –1, 0, 1, 2, 3, ...
4. A whole number (W ) is either a natural number or zero. Thus, the whole number 3 is a natural number while
the whole number 0 is not.
W = 0, 1, 2, 3, ...
5. A natural number (N ) is a non-zero whole number. Thus, the natural number 2 is a non-zero whole number.
N = 1, 2, 3, ...
rATIonAl And IrrATIonAl nuMbErS Ch 1–13

The real numbers are represented graphically as points on a number line as shown below.

Ex. 10. (a) Examine whether 2 is rational or irrational. (b) Show that 2 is not a rational number.
Sol. 1. 4 1 4 2 1 3 5
1 2·00 00 00 00 00 00 00 ...
1
24 1 00

d
96

ite
281 4 00
2 81

m
2824 1 19 00
1 12 96

Li
28282 6 04 00

ny
5 65 64
282841 38 36 00
28 28 41
2828423 10 07 59 00
8 48 52 69
pa
om
28284265 1 59 06 31 00
1 41 42 13 25
C

17 64 17 75
d

∴ 2 = 1.4142135...
An

We observe from the above that 2 is non-terminating and non-repeating. Hence, 2 is not a rational number.
⇒ 2 is an irrational number.
d

(b) To prove that 2 is not a rational number, we will show by contradiction method that it is not a rational
n
ha

number. In this method, we assume that the given number is a rational number and then establish that our supposition
is wrong.
C

Let 2 be a rational number.


p
∴ = 2 , p and q are integers that have no common factor and q ≠ 0.
S

q
Squaring both sides, we get
©

p2
2
= 2 ⇒ p2 = 2q2 ⇒ p2 is an even number
q
⇒ p is also an even number ...(1)
Let p = 2r, r ∈ N ⇒ p = 4r2 2
...(2)
2 2 2 2 2 2
By substituting 2q for p in (2), we get 2q = 4r ⇒ q = 2r
⇒ q is also an even number ...(3)
From (1) and (3), we arrive at the result that p and q are both even numbers.
⇒ p and q are not co-primes. Therefore, our supposition is wrong.
∴ 2 is not a rational number.
⇒ It is an irrational number.
Ch 1–14 ICSE MATHEMATICS–IX

Ex. 11. Show that 3 is not a rational number.


Sol. By finding the square root of 3.
1.7 3 2 0 5 0 8 0 7 ...
1 3·00 00 00 00 00 00 00 00 00 ...
1
27 2 00
1 89
343 11 00
10 29

d
3462 71 00

ite
69 24
346405 1 76 00 00

m
1 73 20 25

Li
34641008 2 79 75 00 00
2 77 12 80 64

ny
3464101607 2 62 19 36 00 00
2 42 48 71 12 49
19 70 64 87 51 pa
om
Since 3 = 1.732050807... is neither terminating nor repeating decimal, therefore, it is not a rational number.
C

By contradiction method.
Suppose 3 is a rational number. Then,
d

p
An

3 = , p and q are integers and have no common factor. Also, q ≠ 0.


q
p2 2 2
Squaring both sides, 3 = ⇒ p = 33q ...(1)
d

q2
n

⇒ p2 is a multiple of 3 ⇒ p is a multiple of 3.
ha

2 2
Let p = 3m3m ⇒ p = 9m ...(2)
Putting this value in (1), we get
C

2 2 2 2
9m = 33q ⇒ q = 3m
9m
S

⇒ q2 is a multiple of 3 ⇒ q is a multiple of 3 ...(3)


(2) and (3) ⇒ p and q both are multiples of 3
©

⇒ 3 is a common factor of p and q.


This contradicts our supposition that p and q have no common factor.
Hence, 3 is not a rational number.
Ex. 12. Identify the following as rational or irriational.
6
(i) 9 (ii) 5 12 (iii) 1.96 (iv) (v) − 0.81
18

( 2 − 3) ( 7 + 1) ( 7 − 1)
2
(vi) 625 (vii) (viii)

(ix) ( 2 + 3)( 7 + 5) (x)


14
2 7
rATIonAl And IrrATIonAl nuMbErS Ch 1–15

Sol. (i) 9 = 3 which is a rational number.

(ii) 5 12 = 5 4 × 3 =
10 3 = 10 × 1.73205080756 ...
= 17.3205080756 ... which is an irrational number.
(iii) 1.96 = 1.4, which is a rational number.
6 1 1 1
(iv) = = = =0.5773502 ... which is an irrational number.
18 3 3 1.7320508 ...

(v) − 0.81 = –0.9 which is a rational number.

(vi) 625 = 25 which is a rational number.

d
( 2 − 3) = 2 + 9 − 6 2 = 11 − 6 2 which is an irrational number.
2

ite
(vii)

( 7 + 1) ( 7 −=
1)
2
( 7 ) − 12 = 7 – 1 = 6 which is a rational number.

m
(viii)
(ix) ( 2 + 3) ( 7 + 5) = 14 + 10 + 21 + 15 , which being the sum of irrational numbers is an

Li
irrational number.

ny
14 7
(x) = = 7, which is an irrational number.
2 7 7

ExErCIsE 1 (b)
(b)
(b
b) pa
om
1. Look at the following real numbers: vi)) There exists a whole number that is not a
((vi
(vi)
3 −2 natural number.
−5, 0, 3, , − 9, 8 , 6.37, π, 4, , 0·03
5 7
C

3. Tell whether each decimal numeral represents a


Tell, rational or an irrational number.
d

(i) which are rational ?


(i) 0.578 (ii) 0.573 333 ...
An

(ii) which are irrational ?


(iii) 0.688 434 445 4... (iv) 0.727 374 75...
(iii) which are positive integers ?
(v) 0.638 754 71... (vi) 0.471 7171...
(iv) which are negative integers ?
d

(vii) 283 (viii) 289.387 000 ...


(v) which number is neither positive nor
n

negative ? (ix) 5.93 (x) 2.30987


ha

2. Write True or False to describe each sentence. (xi) 0.585 885 888...
(i) All rational numbers are real numbers. 4. List three distinct irrational numbers.
C

(ii) All real numbers are rational numbers. 5. Show that (i) 3 (ii) 5 are not rational
S

(iii) Some real numbers are rational numbers. numbers.


(iv) All integers are rational numbers. 6. Is 100 + 36 the same as 100 + 36 ? Give
©

(v) No rational number is also an irrational


reasons.
number.
ANsWErs
3 2
1. (i) −5, 0, , − 9 , 6·37, 4, − , 0·03 (ii) 3, 8, π (iii) 4 (iv) −5, − 9 (v) 0
5 7
2. (i) T (ii) F (iii) T (iv) T (v) T (vi) T
3. (i) rational (ii) rational (iii) irrational (iv) irrational (v) irrational (vi) rational
(vii) rational (viii) rational (ix) rational (x) rational (xi) irrational
4. 2, 3, 5 5. Show by contradiction method 6. No. 100 + 36 = 10 + 6 = 16
100 + 36= 136= 11.662 (approx)
Ch 1–16 ICSE MATHEMATICS–IX

sUrDs
What are surds?
Can numbers like 2, 3, 5, 7, ..., be written as the squares of any rational numbers? The answer is obviously ‘No’.
Can 4 be written as the square of a number? The answer is ‘Yes’. We have (2)2 = 4.
Can 4 be written as the cube of any rational number? The answer is obviously ‘No’. Can 8 be written as the cube
of any rational number. The answer is ‘Yes’. We have (2)3 = 8.

Irrational numbers of the type 2, 3, 5, 17 , ... which are square roots of positive rational numbers that
cannot be expressed as the squares of any rational numbers are called surds.
3
Similarly, 2, 3 4, 3 19, etc., are numbers which are the cube roots of positive rational numbers that cannot be

d
expressed as the cube of any rational numbers and so are surds.

ite
In general, if x is a positive rational number and n is a positive integer, such that x , i.e.,
1/n n
x is irrational,

x is called a surd or a radical.

m
n
then

Li
Ex. 1. State with reason which of the following are surds and which are not :
(i) 98 (ii) 7 × 28 (iii) 3
2 × 3 32 (iv) 15 20 ÷ 4 45 (v) 20 7 ÷ 15 21.

ny
Sol. (i) 98 = 7×7×2 =7 2 , which is not a rational number.
∴ 98 is a surd.
7 × 28 =
pa
7 × 7 × 4 = 7 × 7 × 4 = 7 × 2 = 14, which is a rational number.
om
(ii)
∴ 7 × 28 is not a surd.
C

3
(iii) 2 × 3 32 = 3 2 × 32 = 3
64 = 4 , which is a rational number.
3
∴ 2 × 3 32 is not a surd.
d
An

15 20 15 5 × 4 15 4 15 2 5
(iv) 15 20 ÷ 4 45 == = · · = · = , which is a rational number.
4 45 4 5×9 4 9 4 3 2
15 20 ÷ 4 45 is not a surd.
d


n

20 21 4 3
(v) 20 21 ÷ 15 7 = · = which is not a rational number.
ha

15 7 3
∴ 20 21 ÷ 15 7 is a surd.
C

rationalising
S

When radicals occur in the denominator of a fraction, it is customary to rid the denominators of the radicals. The
process is called rationalising the denominator.
©

Method. The radical in the denominator is multiplied by an appropriate expression, so that the new denominator
contains no radical.
For example,
If Radical is Multiply by To get product free of radicals
7 7 7 × 7= 49= 7
3 3 3
2 4 8= 3 2× 2× 2= 2
2
5+2 5−2 ( 5 ) − 22 = 5 − 4 = 1
11 − 7 11 + 7 ( 11 )2 − 72 =−
11 49 = − 38
2 2
6− 3 6+ 3 ( 6 ) − ( 3) = 6 − 3 = 3
rATIonAl And IrrATIonAl nuMbErS Ch 1–17

rationalisation
Definition. If the product of two irrational numbers is rational, each is called the rationalising factor of the other.
For example, ( a + b ) is the rationalising factor of ( a − b ) as ( a + b ) × ( a − b ) = a − ( b ) = a – b,
2 2 2

which is rational.
Similarly, ( a + b ) is the rationalising factor of a − b as ( a + b ) ( a − b )=
2 2
( a ) − ( b ) = a – b,
which is rational.
The process of multiplication by a rationalising factor is called rationalisation.

Conjugate surds. Two binomial irrational numbers like a + b and a − b which differ only in sign

d
connecting their terms are said to be conjugate. Their main feature is that their product is always rational.

ite
A useful result. If a + b =+
c d , where a and c are rational and b and d are irrational, then a = c and b = d.

simplifying expressions by rationalising the denominator of a fraction

m
Type 1. When the denominator is a monomial irrational number.

Li
1 1 1
n 1−
Rule. If the monomial irrational number is a , i.e., a n , then its rationalising factor is a n, because a n .

ny
1 1 1
1− +1− 1
a n = an n
= a= a, which is a rational number.
Ex. 2. Rationalise the denominator of
1 1 1 pa 1 1
.
om
(i) (ii) (iii) (iv) (v)
7 5 3 3
4 5
36 3
a b5c
4

1
Sol. (i) We know that 7× 7=
7, therefore, to rationalise the denominator of
C

, we multiply both the numerator


7
as well as the denominator by 7.
d
An

1 1 7 7 1
∴ = × = = 7.
7 7 7 7 7
d

1 1 3 1
(ii) = × = 3.
n

5 3 5 3 3 15
ha

1 1 1 2
1−
3
(iii) 4 = 4 3 , so rationalising factor of 4 3 is 4 3
, i.e., 4 3
C

2 1
3
1 1 4 3 (42 ) 3 42 3
16
∴ = × 2= 1 2= =
S

3
4 1
+
3
4
4 3 4 3 43 3 43
©

1 1 1 4
1−
5
(iv) 36 = 36 5 , so rationalising factor of 36 5 is 36 5
, i.e., 36 5
4 1
5
1 1 36 5 (364 ) 5 364
∴ = × 4= = .
5
36 1 1 4
+ 36
36 5 36 5 36 5 5

1 1 1 1 1 1 2 3 4
1− 1− 1−
3 4 5 3 4 5
(v) a b c = a3 b4 c5 , so rationalising factor of a b c is a 3 b 4 c 5
, i.e., a3 b4 c 5

2 3 4 2 3 4 1 1 1
1 1 a3 b4 c5 a3 b4 c5 (a 2 ) 3 (b3 ) 4 (c 4 ) 5 3 a 2 . 4 b 3 . 5 c 4
∴ 3 4 5 = 1 1
× 2 4
=
1 2 1 4 = = .
a b c 1 3 1 3
+ + + a1 b1 c1 abc
a3 b4 c5 a3 b4 c5 a3 3 . b4 4 . c5 5
Ch 1–18 ICSE MATHEMATICS–IX

Ex. 3. Rationalise the denominator and simplify :


26 3 9 16 30
(i) (ii) (iii) (iv) .
13 27 18 5 48
26 26 13 26 13
Sol. (i) = × = = 2 13 .
13 13 13 13
3 3 27 3 27 3 × 9 × 3 3 × 3 3 1
(ii) = × = = = = 3.
27 27 27 27 27 27 3
9 9 18 9 × 9 × 2 9 × 3 × 2 3
(iii) = ×= = = 2.
18 18 18 18 18 2

d
16 30 16 30 48 16 3 × 10 × 16 × 3 16 × 3 × 10 × 4 × 3 16 × 3 × 4 × 10 4
(iv) = × = = = = 10 .

ite
5 48 5 48 48 5 × 48 5 × 48 5 × 48 5

m
Type 2. When the denominator is a binomial irrational number.
Method. Multiply and divide the given irrational number by the conjugate number of the irrational number in the

Li
denominator.
Ex. 4. Rationalise the denominators of the following :

ny
1 2 1 x −3 2 3 +6 7
(i) (ii) (iii) (iv)
((iv
iv)) , x ≥ 0 (v)
2− 3 3− 2 2+ 3 x +3 2− 3

(vi)
5−3 2
(vii)
6+ 3
(viii)
y2 pa
om
5+3 2 9 + 2 18 x2 + y2 + x
1 1 2+ 3 2+ 3 2+ 3
C

Sol. (i) = × = = =2+ 3 .


2− 3 2 − 3 2 + 3 22 − ( 3 ) 2
4−3
d

2 2 3+ 2 2( 3 + 2 ) 2( 3 + 2) 2( 3 + 2)
An

(ii) = × = 2 2
= = = 2( 3 + 2 ) .
3− 2 3− 2 3+ 2 ( 3) − ( 2) 3− 2 1

1 2− 3 2− 3 2− 3
d

(iii) = = = = 3− 2.
( 2 + 3) ( 2 − 3)
n

2+ 3 2−3 −1
ha

( x − 3)
2
x −3 x −3 x −3 x+9−6 x
(iv) = × = = .
C

( x ) − 32 x−9
2
x +3 x +3 x −3

2 3+6 7 2 3+6 7 2 + 3 (2 3 + 6 7 ) ( 2 + 3)
S

(v) = × =2
( 2 ) − ( 3)
2
2− 3 2− 3 2+ 3
©

2 6 + 2 9 + 6 14 + 6 21 2 6 + 2 × 3 + 6 14 + 6 21
= = = − ( 2 6 + 6 + 6 1 4+ 6 2 1) .
2−3 −1
5 − 3 2 5 − 3 2 (5 − 3 2 )
2
5−3 2 25 + 9 × 2 − 30 2 1
(vi) = . = 2 =
= (43 − 30 2) .
5+3 2 5 + 3 2 5 − 3 2 52 − ( 3 2 ) 25 − 18 7

6+ 3 6 + 3 9 − 2 18 ( 6 + 3 ) ( 9 − 2 18 ) 9 6 − 2 108 + 9 3 − 2 54
(vii) = × = =
92 − ( 2 18 )
2
9 + 2 18 9 + 2 18 9 − 2 18 81 − 4 × 18

9 6 −2 3 ×
6 3 + 9 3 − 2 9 × 6 9 6 − 1 23 + 9 3 − 6 6 3 6 − 3 3 6− 3
= = = = .
81 − 72 9 9 3
rATIonAl And IrrATIonAl nuMbErS Ch 1–19

(viii)
y2
=
y2
×
x2 + y 2 − x
=
(
y 2 x2 + y 2 − x )
( )
2
x2 + y 2 + x x2 + y 2 + x x2 + y 2 − x x2 + y 2 − x2

=
y2 ( x2 + y 2 − x )= y ( 2
x2 + y 2 − x
=
) x2 + y2 − x .
2 2 2
x +y −x y2
3 2 4 3 6
Ex. 5. Rationalise the denominator and simplify – + .
3+ 6 6+ 2 2+ 3
3 2 4 3 6
Sol. – +
3+ 6 6+ 2 2+ 3

d
ite
3 2 3− 6 4 3 6− 2 6 2− 3
= × − × + ×
3+ 6 3− 6 6+ 2 6− 2 2+ 3 2− 3

m
3 2 ( 3− 6 ) − 4 3( 6− 2 ) + 6( 2− 3 )
( ) ( ) ( )

Li
= = − 2 3− 6 − 3 6− 2 − 6 2− 3
3−6 6−2 2−3

ny
0.
= − 6 + 12 − 18 + 6 − 12 + 18 =

pa
3 –1
Ex. 6. Find the values of a and b if = a+b 3 .
3 +1
om
( ) ( )
2
3 −1 3 −1 3 −1 3 −1 3 +1− 2 3 4 − 2 3 2 2 − 3
Sol. = × = = = = = 2 − 3.
3 +1 3 +1 3 −1 3 −1 2 2 2
C

3 −1
∴ =2 − 3 =a + b 3 (Given))
d

3 +1
An

∴ Equating rational and irrational parts, we get, a = 2, b = −1.

Type 3. When the denominator is a trinomial irrational number (say of the form p a + q b + r c ).
n d

Method. 1. Multiply the denominator by p a + q b − r c .


ha

2. The denominator will now contain a binomial irrational number which can be rationalised as in Solved
Ex. 5.
C

1
Ex. 7. Rationalise the denominator of .
S

2 + 3 + 10
1 1 2 + 3 − 10 2 + 3 − 10 2 + 3 − 10
©

Sol. = × = =
( 2 + 3 ) − ( 10 )
2 2
2 + 3 + 10 2 + 3 + 10 2 + 3 − 10 2 + 3 + 2 6 −1 0

2 + 3 − 10 2 + 3 − 10 −5 − 2 6
= = ×
−5 + 2 6 −5 + 2 6 −5 − 2 6
( 2+ 3− 1 ) ( −05 − 2 6 ) −5 2 − 2 12 − 5 3 − 2 18 + 5 10 + 2 60
= =
(−5) 2 − ( 2 6 )
2
25 − 24

= −5 2 − 2 4 × 3 − 5 3 − 2 9 × 2 + 5 10 + 2 4 × 15
= −5 2 − 4 3 − 5 3 − 6 2 + 5 10 + 4 15 = −11 2 − 9 3 + 5 10 + 4 15 .
Ch 1–20 ICSE MATHEMATICS–IX

ExErCisE 1 (c)
Simplify :
3−5 5
1 5 46 9. Express in the form ( a 5 − b ) where
1. (i) (ii) (iii) 1 3+ 2 5
3 12 75
224 a and b are simple fractions. (SC)
2. 112 − 63 + 1 2
28 10. Prove that + = 2+ 3.
2 −1 3 +1
4 18 8 75 9 2
3. − + 6 2 4 3 2 6
12 32 3 11. Simplify : − + .
3+ 6 6+ 2 2+ 3
4. Rationalise the denominators of (NDA)

d
1 2
(i) (ii) 12. Simplify :

ite
4− 3 5+ 3 6 6 4 3
(i) + −
1 3+ 2 2 3− 6 3+ 2 6− 2

m
(iii) (iv)
2 5− 3 3− 2
7 3 2 5 3 2

Li
5. Rationalise the denominator and simplify : (ii) − −
10 + 3 6+ 5 15 + 3 2
4+ 5 4− 5 3 2

ny
(i) + (ii) +
4− 5 4+ 5 5− 3 5+ 3 2 1
13. If x = 2 + 3, find the value of x + .

pa
6. Find the values of a and b if x2
3+ 2 1 1 1 2− 3
= a+b 2 = = ×
om
(i) [Hint.
[Hint.
3− 2 x 2+ 3 2+ 3 2− 3
5+2 3 2− 3
= 2− 3
C

(ii) = a+b 3 =
4−3
7+4 3
1
= (2 + 3) + (2 − 3) =
d

∴ x+ 4
7 −1 7 +1 x
An

(iii) − a+b 7
= 2
7 +1 7 −1 2 1  1 2
Now, x + 2 =  x +  − 2 = 4 − 2
x  x 
1
d

7. Rationalise the denominator of .


2+ 3+ 5 = 16 – 2 = 14.]
n

1
2 + 1 , find the value of x 2 +
ha

14. If x = .
8. Taking 2 = 1.414 and 3 = 1.732, find x2
without using tables or long division, the value 5 − 21
C

of 15. If x = , find the value of


2
1 2 1 1
S

(i) (ii) (SC) (i) x + and


2
(ii) x + 2 .
3− 2 3− 2 x x
©

ANsWErs
3 1 11
1. (i) (ii) 15 (iii) 3 2. 17 7 3. 0
3 6 15
4+ 3 2 5+ 3
4. (i) (ii) 5− 3 (iii) (iv) 5 + 2 6
13 17
42 25 + 3 11 6 2
5. (i) (ii) 6. (i)= a = ,b (ii) a = 11, b = –6 (iii) a = 0, b = −
11 22 7 7 3
2 3+3 2 − 3 0 21 59
7. 8. (i) 0.631 (ii) 6.292 (approx) 9. 5− 11. 3 2 − 6
12 11 11
12. (i) 0 (ii) 1 14. 6 15. (i) 5 (ii) 23
rATIonAl And IrrATIonAl nuMbErS Ch 1–21

ChAPTEr TEsT
1. A number is an irrational number if and only if its decimal representation is
(a) non-terminating (b) non-terminating and repeating
(c) non-terminating and non-repeating (d) terminating
2. Which of the following is an irrational number?
(a) 29 (b) 441 (c) 0.5948 (d) 5.318
3. (−2 − 3) (−2 + 3) when simplified is
(a) positive and irrational (b) positive and rational
(c) negative and irrational (d) negative and rational
4. If x 10, then the value of x is
6 × 15 =

d
(a) 3 (b) ± 3 (c) 3 ((dd)
(d)d) 6

ite
2 5
5. Two rational numbers between and are

m
7 7
1 2 1 3 3 3 5 8
and and and

Li
(a) and (b) (c) (d)
14 14 2 2 14 7 14 14
5 7

ny
6. An irrational number between and is
7 9

pa
(a) 0.75 (b) 6 (c) 0.7507500075000... (d) 0.7512
7
7. If 2 = 1.4142, then the value of
om
correct to two decimal places is
3+ 2
(a) 1.59 (b) 1.60 ((cc)) 2.58
(c) (d) 2.57
C

1
8. Taking 3 as 1.732 and 2 = 1.414, the value of is
3+ 2
d

(a) 0.064 (b) 0.308 (c) 0.318 (d) 2.146


An

 1
x
9. If = 3 + 2, then the value of  x +  is
 x
(a) 2 ((bb)) 3
(b) (c) 2 2 (d) 2 3
d

1
n

10. If x= 2 + 3, then the value of x + is


ha

x
(a) 3 + 3 (b) 6 (c) 2 6 (d) 6
C

ANsWErs
S

1. (c) 2. (a) 3. (b) 4. (a) 5. (d) 6. (c) 7. (a)


8. (c) 9. (d) 10. (b)
©
©
S
C
ha
nd
An
d
C
om
pa
ny
Li
m
ite
d
UNIT 2 COMMERCIAL MATHEMATICS

2
CHAPTER COMPOUND INTEREST

d
ite
Introduction

m
Suppose a man who deposits ` 1000 in a savings account at an interest rate of 10% per annum. At the end of one year
he will get ` 100 interest on his deposit. However, unless he takes out his ` 100 in cash, it will be added to his original

Li
` 1000. Thus, if he leaves his money in his account, in the next year the bank will be paying him interest on his original

ny
` 1000 plus the ` 100 interest, i.e., ` 1100. In the third year the interest will once again be added to the new principal of
` 1100, and so on for as long as the money is left in the account.

pa
This kind of interest is known as compound interest.. Some banks and other institutions pay interest yearly, others
every six or even three months.
om
Type 1. When time is in whole number of year.
Ex. 1. Find the compound interest on ` 8500 for 2 year at 8% per annum.
C

Sol. Step 1. Principal for the first year = ` 8500


8500 × 8 × 1 P× R ×T
d

Interest for the first year = ` = ` 680 I=


100 100
An


∴ Amount at the end of first year = ` 8500 + ` 680 = ` 9180
Step 2. Principal for the second year = ` 9180
n d

9180 × 8 × 1
Interest for the second year = ` = ` 734.40
ha

100
∴ Amount at the end of second year = ` 9180 + ` 734.40 = ` 9914.40.
C

∴ Compound interest after 2 years = ` 9914.40 – ` 8500 = ` 1414.40.


Ex. 2. Find the compound interest when principal = ` 50,000, rate = 10% p.a. and time = 3 years. Find the amount
S

payable at the end of 3 years.


©

Sol. Step 1. Principal for the first year = ` 50, 000, Rate = 10% p.a.

50000 × 10 × 1 P× R ×T
Interest for the first year = ` = ` 5,000 I=
100 100
∴ Amount at the end of the first year = ` (50000 + 5000) = ` 55,000
Step 2. ∴ Principal for the second year = ` 55,000
55000 × 10 × 1
Interest for the second year = ` = ` 5,500
100

Amount at the end of second year = ` (55000 + 5500) = ` 60,500


Ch 2–2 ICSE MATHEMATICS–IX

Step 3. ∴ Principal for the third year = ` 60,500


60,500 × 10 × 1
∴ Interest for the third year = ` = ` 6,050
100
Amount at the end of third year = ` (60,500 + 6,050) = ` 66,550
∴ Compound Interest = ` 66,550 – ` 50,000 = ` 16,550.
Type 2. When the time is not whole number of years.
1
The C.I. is reckoned yearly but the time may be some years and some months, like 2 years, 1 year 4 months,
2
1
2 years 3 months, i.e., 2 years etc.
4

d
ite
1
Ex. 3. Find the compound interest for 2 years on ` 10,000 lent at 5% p.a. reckoned annually.
2

m
P ×T × R
Sol. Interest for 1st year : Principal (P) = ` 10,000; T = 1 year; R = 5% p.a.

Li
100
10,000 × 1 × 5
∴ Interest = ` = ` 500

ny
100
Amount = Principal + Interest
= ` 10,000 + ` 500 = ` 10,500
Interest for 2nd year : P = ` 10,500; T = 1 year, R = 5% p.a. pa
om
10,500 × 1 × 5
∴ Interest = ` = ` 525
100
C

Amount = Principal + Interest = ` 10,500 + ` 525 = ` 11,025


d

1 1
An

Interest for last year : P = ` 11,025; T = year; R = 5% p.a.


2 2
1
11025 × × 5
d

2 55125 27562.5
∴ Interest = ` = `= ` = ` 275.625
n

100 200 100


ha

= ` 275.63 (approx.)

C

Amount = Principal + Interest


= ` 11025 + ` 275.63 = ` 11300.63
S

Hence, Required compound interest = Amount – Principal


= ` 11300.63 – ` 10000 = ` 1300.63.
©

Ex. 4. Find the compound interest for 1 year 3 months on ` 6000 lent at 8% p.a., reckoned annually.
Sol. Interest for 1st year : P = ` 6000, T = 1 year, R = 8% p.a.
6000 × 1 × 8
∴ Interest = ` = ` 480
100
Amount = Principal + Interest = ` 6000 + ` 480 = ` 6480.
1 1
Interest for the next 3 months, i.e., year : P = ` 6480, T = year, R = 8% p.a.
4 4
1
6480 × × 8
4 12960
∴ Interest = ` = ` = ` 129.60
100 100
CoMpound InTErEST Ch 2–3

⇒ Amount = ` 6480 + ` 129.60 = ` 6609.60


∴ Compound interest = Amount – Principal
= ` 6609.60 – ` 6000 = ` 609.60.
Type 3. When the interest is different for sucessive years.
Ex. 5. Calculate the amount and the compound interest on ` 20,000 for 3 years when the rates of interest for
successive years are 6%, 8% and 10% respectively.
Sol. For 1st year : P = ` 20,000, T = 1 year, R = 6% p.a.
20,000 × 1 × 6
∴ Interest = ` = ` 1200
100
Amount = Principal + Interest = ` 20,000 + ` 1200 = ` 21,200

d
For 2nd year : P = ` 21,200, T = 1 year, R = 8% p.a.

ite
21, 200 × 1 × 8
∴ Interest = ` = ` 1696

m
100
⇒ Amount = Principal + Interest = ` 21,200 + ` 1696 = ` 22,896

Li
For 3rd year : Principal = ` 22,896, Time = 1 year, R = 10% p.a.
22,896 × 1 × 10

ny
∴ Interest = ` = ` 2289.60
100

Hence, pa
Amount = ` 22,896 + ` 2289.60 = ` 25,185.60
Compound interest = Amount – Principal
om
= ` 25,185.60 – ` 20,000 = ` 5185.60.
Type 4. When the interest is paid half-yearly.
C

1
Ex. 6. Find the amount and the compound interest on ` 24000 for 1 year at 10% per annum, the interest
d

2
being compounded half-yearly.
An

Sol. Rate of interest per annum = 10%


1
⇒ Rate of interest half yearly =
d

of 10% = 5%
2
n

1
Number of half-yearlies in 1 years = 3
ha

2
Interest for 1st half-year : P = ` 24000, T = 1 half year, R = 5% per half year.
C

24000 × 1 × 5
∴ Interest = ` = ` 1200
S

100
∴ Amount after the first half-year = ` 24000 + ` 1200 = ` 25200
©

Interest for 2nd half-year : P = ` 25200, T = 1 half year, R = 5% per half year
25200 × 1 × 5
∴ Interest for 2nd half year = ` = ` 1260
100
∴ Amount after 2nd half year = ` 25200 + ` 1260 = ` 26460
Interest for 3rd-half year : P = ` 26460, T = 1 half year, R = 5% per half year
26460 × 1 × 5
∴ Interest for 3rd half year = ` = ` 1323
100
∴ Amount after 3rd half year = ` 26460 + ` 1323 = ` 27783
⇒ Required compound interest = Amount – Principal
= ` 27783 – ` 24000 = ` 3783.
Ch 2–4 ICSE MATHEMATICS–IX

ExErCisE 2 (a)

Find the compound interest on the following : (iv) Find the compound interest at the end of
Principal Rate% p.a. Number of years 3 years.
1. ` 10000 12% 2 10. Shanker takes a loan of ` 10, 000 at a compound
interest rate of 10% per annum (p.a.)
2. ` 5000 10% 2
(i) Find the compound interest after one year.
1
3. ` 2800 10% 1 (ii) Find the compound interest for 2 years.
2
(iii) Find the sum of money required to clear the
4. ` 2000 20% 2 debt at the end of 2 years.
1

d
5. ` 20480 6 % 2 years 73 days (iv) Find the difference between the
the compound

ite
4 interest and the simple interest at the same
73
[Hint. 2 years 73 days = 2 years rate for 2 years.

m
365 11. Find the compound interest on ` 5000 at
1 1

Li
= 2 years = 2 years + year] 12% p.a. for 1 year, compounded half-yearly.
5 5 12. Find the amount and the compound interest on

ny
6. Find the amount and compound interest on a sum of 1
` 16000 for 1 years at 10% p.a., the interest
` 15625 at 4% per annum for 3 years compounded 2

7.
annually.
To renovate his shop, Anurag obtained a loan of pa
being compounded half-yearly.
13. Calculate the amount due and the compound
om
` 8000 from a bank. If the rate of interest at 5% interest on ` 40000 for 2 years when the rate
per annum is compounded annually, calculate the of interest for successive years is 7% and 8%
C

compound interest that Anurag will have to pay respectively.


after 3 years.
14. If the simple interest on a sum of money for
d

8. Maria invests ` 93750 at 9.6% per annum for 2 years at 5% per annum is ` 50, what will be the
An

3 years and the interest is compounded annually. compound interest on the same sum at the same
Calculate: rate for the same time.
(i) the amount standing to her credit at the end of [Hint. First find P by using
d

the second year,


n

P ×T × R P× 2×5
(ii) the interest for the 3rd year. ⇒ 50 =
ha

I= ]
100 100
9. A sum of ` 9, 600 is invested for 3 years at 10%
p.a. compound interest. 15. A man invests ` 46,875 at 4% per annum compound
C

(i) What is the sum due at the end of the first year ? interest for 3 years. Calculate:
S

(ii) What is the sum due at the end of the second (i) the interest for the 1st year;
year ? (ii) the amount standing to his credit at the end
©

of the 2nd year;


(iii) Find the compound interest earned in the first
2 years. (iii) the interest for the 3rd year. (ICSE)

ANsWErs
1. ` 2,544 2. ` 1, 050 3. ` 434 4. ` 880 5. ` 2929
6. ` 17576; ` 1951 7. ` 1261 8. 112614 ; ` 10810.94
9. (i) ` 10560 (ii) ` 11616 (iii) ` 2016 (iv) ` 3177.60
10. (i) ` 1000 (ii) ` 2100 (iii) ` 12100 (iv) ` 100
11. ` 618 12. Amount = ` 18,522; C.I. = ` 2,522 13. Amount = ` 46,224; C.I. = ` 6224
14. ` 51.25 15. (i) ` 1,875 (ii) ` 50,700 (iii) ` 2,028
CoMpound InTErEST Ch 2–5

Compound interest formula


The arithmetic involved in working out compound interest can be quite complicated when the number of payments,
or the length of time, or both are large.
However, just as there is a formula for calculating simple interest, so also there is a formula which can be used to
work out compound interest. Using this formula saves a great deal of time.
The formula is,
n
 r 
A = P 1 + 
 100 
where A is the amount, P is the principal, r is the rate per cent per annum, n is the time in year.

d
Type 1.

ite
Ex. 1. Find the amount and interest of ` 5000 in 2 years at 10% per annum compound interest.
Sol. P = ` 5000, R = 10%, n = 2 years

m
n
 r 

Li
A (Amount) = P 1 + 
 100 

ny
2
 10 
= ` 5000 1 + 
 100 
110 110
×
= ` 5000 × = ` 6050 pa
om
100 100
∴ Compound interest = A – P = ` 6050 – ` 5000 = ` 1050.
C

Type 2. When the Interest is compounded half-yearly.


1
d

Ex. 2. Find the amount of ` 256 in one year at 12 % per annum, when the interest is compounded
2
An

half-yearly.
1 25
Sol. P = ` 256, 1 year = 2 half years, ∴ Annual rate = 12 % = % , n = 2 half years
2 2
n d

1  25  25
∴ Half-yearly rate =  % = %
ha

2  2  4
2
 25 
C

n
 r   4 
A) = P 1 +
Thus, Amount (A)
((A  = ` 256  1 + 
 100   100 
S

2
 1
©

= ` 256 1 + 
 16 
17 17
= ` 256 × × = ` 289.
16 16

Ex. 3. Sunil loaned ` 8192 to Ravi to enable him to purchase a T.V. set. If Sunil charged interest at the rate of
1
12.5% per annum, compounded half-yearly, calculate the amount that Sunil will pay to Ravi after 1 years.
2
25 1  25  25
Sol. Principal (P) = ` 8192, Rate (r) = 12.5% = % yearly =   = % half yearly,
2 2 2  4
1 3 3
Time (n) = 1 years = years = × 2 = 3 half years
2 2 2
Ch 2–6 ICSE MATHEMATICS–IX

n 3 3
 r   25 4   1
∴ Amount (A) = P 1 +  = ` 8192  1 +  = ` 8192 1 + 
 100   100   16 
3
 1 17 17 17
= ` 8192 1 +  = ` 8192 × × × = ` 9826.
 16  16 16 16
1
Hence, amount paid by Sunil to Ravi after 1 years = ` 9826.
2
Type 3. When the rate of interest for successive years are different.
If the rate of interest is different for every year say, r1, r2, r3, for the first, second, third year, then the amount after
3 years is given by
 r  r  r 
A = P 1 + 1  1 + 2  1 + 3 

d
 100   100   100 

ite
Ex. 4. Find the compound interest on ` 80,000 for 3 years if the rates for the 3 years are 4%, 5% and 10%

m
respectively.
 4  5  10  104 105 110

Li
Sol. Amount = ` 80,000 1 +  1 +  1 +  = ` 80,000 × × ×
 100   100   100  100 100 100

ny
= ` 96,096
∴Compound interest = ` 96,096 – ` 80,000 = ` 16,096.

Population problems
pa
om
n
 r 
The formula A = P 1 +  is called the compound interest law, and applies to any quantity which increases or
 100 
C

decreases so that the amount at the end of each period of constant lengths bears a constant ratio to the amount at the
beginning of that period. This ratio is called the growth factor, if it is greater than 1, and the decay factor, if less than 1.
d

For example, if the population of a town increases steadily by 2% p.a. of the population at the beginning of each
An

 2 
year, the yearly growth factor is 1 + n
 , i.e., 1.02, and the population after n years is (1.02) times the population at
 100 
2 
d


the beginning of that period. If the population decreases by 2%, then the yearly decay factor is 1 −  , i.e., 0.98.
n

 100 
ha

Ex. 5. If the population of a town decreases 6.25% annually and the present population is 20480000, find its
population after 3 years.
C

n
 d 
Sol. Required population = P 1 −  , Here d = 6.25
 100 
S

3 3
 6.25   625 
∴ Population after 3 years = 20480000 1 −  = 20480000 1 − 
©

 100   10000 
3
 1 15 15 15
= 20480000 1 −  = 20480000 × × × = 16875000.
 16  16 16 16
Ex. 6. The population of a certain city is 125000. If the annual birth rate is 3.3% and the annual death rate is
1.3%, calculate the population after 3 years.
Sol. Present population of the city (P) = 125000
Time (n) = 3 years, Rate of birth (R1) = 3.3%, Rate of death (R2) = 1.3%.
So, the net rate of increase (R1 – R2) = 3.3 – 1.3 = 2%
3
 2 
∴ Population after 3 years = 125000 1 +  = 125000 × (1.02)
3
 100 
= 125000 × 1.061208 = 132651 = 132651.
compound interest Ch 2–7

ExErCisE 2 (b)

Calculate the amount and the compound interest by [Given that (1.0575)40 = 9.35869]
using the formula for compound interest.
[Hint. Sol. Principal (P) = ` 5000,
Principal Rate of interest Time (in years)
1. ` 625 4% p.a. 2 11.5
R = 11.5% p.a. = % = 5.75% half yearly
2. ` 8000 15% p.a. 3 2
3. ` 1000 10% p.a. 3
1 Compounded half yearly,
4. ` 8000 10%half-yearly 1 Time (n)) = 2008 – 1988 = 20 years = 40 half years.
2
1 Amount of bonds on maturity of

d
5. ` 700 20% half yearly 1
2 n

ite
 r 
6. Sangeeta lent ` 40960 to Amar to purchase a shop A = P 1 +  = P (1 + r × 0.01)
n
 100 

m
at 12.5% per annum. If the interest is compounded
semi-annually, find the interest paid by Amar after i.e., A = ` 5000 (1 + 5.75 × 0.01)40

Li
1 = ` 5000 (1.0575)40
1 years.
2 = ` 5000 × 9.35869

ny
7. Sudhir lent ` 2000 at compound interest at 10% = ` 46793.45000 = ` 46793.45
payable yearly, while Prashant lent ` 2000 at
pa
Hence, the amount that he gets on maturity
compound interest at 10% payable half-yearly. = ` 46793.45.]
Find the difference in the interest received by
om
11. A district contains 64000 inhabitants. If the
Sudhir and Prashant at the end of one year.
1
8. How much will ` 25000 amount to in 2 years at population increases at the rate of 2 % per
C

2
compound interest, if the rates for the successive
annum, find the number of inhabitants at the end
years be 4 and 5 per cent per year ?
d

of 3 years.
9. Umesh set up a small factory by investing
An

12. The Nagar Palika of a certain city started campaign


` 40,000. During the first three successive years to kill stray dogs which numbered 1250 in the city.
his profits were 5%, 10% and 15% respectively. If As a result, the population of stray dogs started
d

each year the profit was on previous year’s capital, decreasing at the rate of 20% per month. Calculate
calculate his total profit.
n

the number of stray dogs in the city three months


ha

Public Undertakings after the campaign started.


10. Himachal Pradesh State Electricity Board issued in 13. 8000 blood donors were registered with a charitable
C

July 1988, 20 year bonds worth ` 6.25 crore. The hospital. Some student organizations started
mobilizing people for this noble cause. As a result,
issue price of each bond is ` 100 and it carries an
the number of donors registered, increased at the
S

annual interest of 11.5%, compounded half-yearly.


rate of 20% per half year. Find the total number of
Jasbir invested ` 5000 in these bonds. Find the
©

1
amount that he gets on maturity of the bonds in new registrants during 1 years.
2008. 2

ANsWErs

1. ` 676; ` 51 2. ` 12,167; ` 4,167 3. ` 1331; ` 331


4. ` 9261; ` 1261 5. ` 931.70, ` 231.70 6. ` 8170
7. ` 5 8. ` 27,300 9. ` 13,130
10. ` 46793.45 11. 68, 921 12. 640 13. 5824
Ch 2–8 ICSE MATHEMATICS–IX

inverse problems on compound interest


(A) To find the principal
Case 1. When amount, time and rate are given.
Ex. 1. Find what sum will amount to ` 73810 in two years at 10% per annum compound interest.
2
 10  11 11 100
Sol. 73810 = P 1 +  =P× × ⇒ P = 73810 × = 61,000
 100  10 10 121
Hence, the required sum is ` 61,000.
1 1
Ex. 2. What sum will become ` 4913 in 1 years if the rate of interest is 12 % compounded half-yearly ?
2 2

d
ite
25 25 3
Sol. Amount (A) = 4913, rate (r) = % p.a. = % half-yearly, time (t) = years = 3 half years
2 4 2

m
n
r 

Li

∴ Using the formula, A = P 1 +  , we have
 100 

ny
3 3 3
 25 / 4   25   1
4913 = P 1 +  =P 1 +  =P 1 + 

pa
 100   400   16 
17 17 17 16 16 16
4913 = P × × × × ×
om
⇒ P = 4913 × = 4096
16 16 16 17 17 17
Hence, the required sum = ` 4096.
C

Case 2. When the difference between compound interest and simple interest for a certain period is given.
d

Ex. 3. The difference between the simple and compound interest for a certain sum of money for 3 years at 5%
An

per annum is ` 122.00. Find the sum.


Sol. Let the sum be ` 100.
d

100 × 3 × 5
∴ S.I. on ` 100 for 3 years @ 5% p.a. = ` = ` 15
n

100
ha

3
 5 
Amount at C.I. on ` 100 for 3 years @ 5% p.a. = ` 100 1 + 
C

 100 
105 105 105 9261
S

= ` 100 × × × =`
100 100 100 80
©

1261 61
Difference of C.I. and S.I. = ` – ` 15 = `
80 80
61
If the difference is ` , sum = ` 100
80
80
∴If the difference is ` 122, sum = 100 × × 122 = ` 16000.
61
(B) To find the rate of interest

Ex. 4. Savita invested ` 1000 in a finance company and received ` 1331 after 3 years, Find the rate of interest
per cent per annum compounded annually.
Sol. Principal (P) = ` 1000, Amount (A) = ` 1331, Time (n) = 3 years
CoMpound InTErEST Ch 2–9

n
 r 
A = P 1 + 
 100 
3 3 3
 r   r  1331  11 
∴ 1331 = 1000 1 +  ∴ 1 +  = = 
 100   100  1000  10 
r 11 r 1
∴ 1+ = ⇒ =
100 10 100 10
1
∴ r= × 100 = 10
10
Hence, the required rate of interest is 10% p.a.

d
Ex. 5. Preeti purchased six years National Savings Certificate for ` 1000. After six years she got ` 2015. Find

ite
1/12
2.015)) = 1.06012].
the rate of interest, if the interest is compounded half-yearly. [Given that (2.015)
2.015

m
Sol. Principal (P) = ` 1000, Amount (A) = ` 2015,Time (n)) = 6 years. When the interest is compounded half-yearly,
then the time is 12 half - years. Let the half-yearly rate of interest be r%.. Then,

Li
n 12
 r   r 
A = P 1 +  ∴ 2015 = 1000 1 + 

ny
 100   100 
12
 r  2015
⇒ 1 +
 100 
 =
1000
= 2.015
pa
om
 r 
⇒ 1 +
1/12
 = (2.015) = 1.06012
 100 
C

r
⇒ = 1.06012 – 1 = 0.06012
100
d

⇒ r = 0.06012 × 100 = 6.012


An

∴Rate of interest per annum = 2r % = 2 × 6.012% = 12.024% ≅ 12 %


∴Hence, the required rate of interest is 12% p.a.
d

Ex. 6. The difference between the compound interest and the simple interest on ` 42000 for two years is
n

` 105 at the same rate of interest per annum. Find :


ha

(i) the rate of interest, (ii) the compound interest earned in the second year.
Sol. (i)) Let the rate of interest be r % p.a. It is given that principal (P) = ` 42000 and time (n) = 2 years
C

P× T × R 42000 × 2 × r
∴ S.I. = =` [Do not simplify]
100 100
S

 n   2   r  2 2r 
r  r 
C.I. = ` P 1 + − 1  1 + − 1  +
©

  = ` 42000   = ` 42000  
 100    100    100  100 
 r  2 2r  2r
Difference of C.I. and S.I. = ` 42000   +  – ` 42000 ×
 100  100  100
2
 r  21 2
= ` 42000 ×   =` r
 100  5
But the difference is given to be ` 105.
21 2 5
∴ r = 105 ⇒ r2 = 105 × = 25 ⇒ r = ±5, leaving out the –ve value, we have r = 5
5 21
Hence, the rate of interest is 5% p.a.
Ch 2–10 ICSE MATHEMATICS–IX

 2 
5   441  41
(ii) C.I. = ` 42000 1 +  − 1 = ` 42000  − 1 = ` 42000 × = ` 4305
 100    400  400

∴ Interest earned in 1 year = ` 4305 – ` 2100 = ` 2205.


(C) To find the time
Ex. 7. The compound interest on ` 30,000 at 7% per annum for a certain time is ` 4347. Find the time.
Sol. Amount (A) = ` 30 000 + ` 4347 = ` 34347, Principal (P) = ` 30 000, rate (r) = 7%. Let the time be t years.
t
 1 
Then, A = P 1 + 
 100 

d
t t
 7   107 

ite
∴ 34347 = 30000 1 +  = 30000 ×  
 100   100 

m
t 2
 107  34347 11449  107 
⇒   = = =   ∴ t=2
 100  30000 10000  100 

Li
Hence, the required time is 2 years.

ny
ExErCisE 2 (c)

pa
8. Hari purchased Relief Bonds for ` 1000, a sum
1. What amount of money should Mohan invest in
a bank in order to get ` 1323 in 2 years at 5% which will fetch him ` 2000 after 5 years. Find
om
compounded annually ? the rate of interest if the interest is compounded
half-yearly.
2. Find the sum which amounts to ` 1352 in 2 years 10
at 4% compound interest. [Given that 2 = 1.072]
C

3. What Principal will amount to ` 9768 in two years, 9. ` 8000 became ` 9261 in a certain interval of time
at the rate of 5% per annum, C.I. Find the time.
d

if the rates of interest for the successive years are


10. In how many years will a sum of ` 3000 at 20%
An

10% p.a. and 11% p.a. respectively.


per annum compounded semi-annually become
4. On what sum of money does the difference between ` 3993?
the simple interest and compound interest in 2 years 11. A sum of money put out at compound interest
d

at 5% per annum is ` 15 ? amounts in 2 years to ` 578.40 and in 3 years to


n

5. The difference between simple and compound ` 614.55. Find the rate of interest.
ha

2 2
terest on the same sum of money at 6 % for
interest  r 
3 [Hint: 578.40 = P 1 + 
 100 
C

3 years is ` 184. Determine the sum.


3
6. On what sum of money will the difference between  r 
S

the simple interest and the compound interest for = P 1 +


614.55 
2 years at 5% per annum be equal to ` 50 ?  100 
©

7. Find the rate per cent per annum, if compounded 614.55 r 


= 1+
100 
Dividing,
yearly 578.40
(i) Principal = ` 196, Amount = ` 225, 25
12. A sum compounded annually becomes times
time, = 2 years. 16
(ii) Principal = ` 3136, Compound interest = ` 345, of itself in 2 years. Determine the rate of interest
Time = 2 years. per annum.

ANsWErs
1. ` 1200 2. 1250 3. ` 8000 4. ` 6000 5. ` 13,500 6. ` 20,000
1 5 1 1
7. (i) 7 % (ii) 5 % 8.14.4% 9. 3 years 10. 1 years 11. 6 % 12. 25%
7 14 2 4
compound interest Ch 2–11

Growth and depreciation


n
 r 
The formula A = P 1 +  is called the compound interest law, and applies to any quantity which increases or
 100 
decreases, so that amount at the end of each period of constant length bears a constant ratio to the amount at the beginning
of that period. This ratio is called the growth factor, if it is greater than 1, and the decay factor if less than 1.
For example, if the population of a town increases steadily by 2% p.a., of the population at the beginning of each
 2 
year, the yearly growth factor is 1 +
n
 , i.e., 1.02 and the population after n years is (1.02) times the population at
 100 
the beginning of the period.
Similarly, if the value of the machinary in a factory depreciates steadily by 10% p.a., of its value at the beginning

d
10
of each year, the yearly decay factor is 1 − , i.e., 0.9, and the value after n years is (0.9)n times its value when new.

ite
100
Assuming that the rate of growth or depreciation is constant, the formulae for growth and depreciations are similar

m
to the formulae for compound interest and can be stated as under:

Li
A. For growth.
n
 r 
V = V0  1 +  , where V0 is the present value of an item and V its value after n years.

ny
1.
 100 

pa
Value after Present value
n years
If V0 was the value of the item n years ago and V is its present value, then also we have
om
n
 r 
Present value → V = V0  1 + 
 100 
C

Value n years ago


 r  r 
2. V = V0  1 + 1   1 + 2  , if the rate of growth is r1% during the first year (or unit of time) and r2% during
d

 100   100 
An

the second year (or unit of time).


Here, V0 is the present value of the item and V its value after 2 years.
d

B. For depreciation.
n

n
 r 
V = V0  1 −
100 
ha

1.

C

Depreciated value Present value


after n years n
 r 
S

2. V = V0  1 − 
 100 
©

Present Value n years ago


value

Note : The period during which an item can be effectively used is called its life span, after which it is sold as
waste or scrap.
1
Ex. 1. A property decreases in value every year at the rate of 6 per cent of its value at the beginning of that
4
year. If its value at the end of 3 years be ` 21093.75, what was its worth at the beginning of these three
years ?
Sol. Let V3 be the depreciated value for the property at the end of 3 years and V0 the value at the beginning of these
3 years, then
n
 r 
V3 = V0 1 − 
 100 
Ch 2–12 ICSE MATHEMATICS–IX

3
 1
 6  3
3 84375  15 
⇒ 21093 = V0 1 − 4  ⇒ V0  
=
4  100  4  16 
84375 16 16 16
⇒ V0 = × × × = 25,600
4 15 15 15
Hence, the property was worth ` 25,600 at the beginning of the three years.
Ex. 2. The value of a car is ` 1,00,000 at present. If the value depreciates 10% in the first year, 8% in the
2nd year and 5% in the 3rd year, what will be its depreciated value after 3 years ?
Sol. Let V3 be the depreciated value of the car after 3 years, then

d
 10   8  5 

ite
V3 = 1,00,000 1 −  1 −  1 − 
 100  100  100 

m
9 23 19
= 1,00,000 × × × = 78,660

Li
10 25 20
Hence, the depreciated value of the car after 3 years will be ` 78,660.

ny
Ex. 3. If the population of a town decreases 6.25% annually and the present population is 2,04,80,000 find its
population after 3 years.


Sol. Required population = P 1 −
d 
n
pa
om
 . Here d = 6·25
 100 
3 3
 6·25   1
∴ Population after 3 years = 2,04,80,000 1 − =  2,04,80,000 1 − 
C

 100   16 
15 15 15
d

= 2,04,80,000 × × × = 1,68,75,000.
16 16 16
An

Ex. 4. Neha started a business with an initial investment of ` 5,00,000. In the first year, she incurred a loss
of 4%. However, during the second year, she earned a profit of 5% which in the third year rose to 10%.
d

Calculate the net profit for the entire period of three years.
n

 4  5  10 
ha

Sol. Net amount obtained after 3 years = ` 5,00,000 1 −  1 +  1 + 


 100   100   100 
C

24 21 11
= ` 5,00,000 × × × = ` 5,54, 400
25 20 10
S

∴ Net Profit = ` 5,54,400 – ` 5,00,000 = ` 54,400.


©

Ex. 5. The present price of a scooter is ` 7,290. If its value decreases every year by 10%, then what was its value
3 years ago ?
Sol. Let the value of the scooter three years ago be ` P.
3
 10 
Then its present value is ` P 1 − .
 100 
But, it is given that the present value = ` 7,290
3
 10 
∴ P 1 − 7290
 =
 100 
CoMpound InTErEST Ch 2–13

3 3
 1 9
⇒ P 1 −  = 7290 ⇒ P   = 7290
 10   10 
729 P 7290 × 1000
⇒ = 7290 ⇒P
= = ⇒ P 10000
1000 729
∴ The value of the scooter 3 years ago was ` 10,000.
Ex. 6 Ashish purchased a boat for ` 16,000. If the cost of the boat after two years depreciates to ` 14,440, find
the rate of depreciation.
Sol. Let the rate of depreciation be R% p.a.
2 2
 R  14440  R 

d
14440 = 16000 1 −  ⇒ 1 −
= 
 100  16000  100 

ite
2 2 2
361  R   19   R 

m
⇒ =1 −  ⇒   = 1 − 
400  100   20   100 

Li
19 R R 19 R 1
⇒ 1−
= ⇒ 1−
= ⇒ = ⇒R=
5

ny
20 100 100 20 100 20
∴ Rate of depreciation = 5% p.a.

ExErCisE 2 (d pa
d)
d)
((d)
om
1. The cost of a machine depreciates by 10% every year. count of the bacteria in a sample is 10,000, find the
If its present worth is ` 18,000; what will be its value bacteria count at the end of 3 hours.
after 3 years?
C

7. The production of refrigerators in a factory rose from


2. The population of a town increases by 20% every 40000 to 48400 in 2 years. Find the rate of growth
d

year. If its present population is 2,16,000. Find its p.a.


An

population (i) after 2 years, (ii)) 2 years ago. 8. The value of a flat worth ` 5,00,000 is depreciating
3. The machinery of a certain factory is valued at at the rate of 10% p.a. In how many years will its
` 18,400 at the end of 1980. If it is supposed to value be reduced to ` 3,64,500 ?
d

depreciate each year at 8% of the value at the


9. Rachit bought a flat for ` 10 lakh and a car for
n

beginning of the year, calculate the value of the


` 3,20,000 at the same time. The price of the flat
ha

machine at the end of 1979 and 1981. (ICSE)


appreciates uniformly at the rate of 20% p.a.,
4. The present value of a scooter is ` 15,360. If its value while the price of the car depreciates at the rate of
C

1 15% p.a. If Rachit sells the flat and car after 3 years,
depreciates 12 % every year, find its value after
2 what will be his profit or loss ?
S

3 years.
5. A new car is purchased for ` 2,50,000. Its value 10. 8,000 workers were employed by a company to
©

depreciates at the rate of 10% in the first year, 8% in complete a job in 4 years. At the end of first year,
2nd year and then 6% every year. Find its value after 5% of the workers were retrenched. At the end of
4 years. second year, 5% of those working at that time were
retrenched. However to complete the job in time, the
6. The bacteria in a culture grows by 10% in the first number of workers was increased by 10% of those
hour, decreases by 10% in the second hour and again working at the end of third year. How many workers
increases by 10% in the third hour. If the original were working during the fourth year?

ANsWErs
1. ` 13,122 2. (i) 3,11,040 (ii) 1,50,000 3. ` 20,000; ` 16,928 4. ` 10,290
5. ` 1,82,905.20 6. 10,890 7. 10 % p.a. 8. 3 years
9. Net profit = ` 6,04,520 10. 7,942
Ch 2–14 ICSE MATHEMATICS–IX

ChAPtEr tEst
1. Nikita invests ` 6,000 for two years at a certain rate Multiple Choice Questions (MCQs)
of interest compounded annually. At the end of first 6. A sum of ` 12,000 deposited at compound interest
year it amounts to ` 6,720: Calculate: becomes double after 5 years. After 20 years, it will
(i) the rate of interest become
(ii) the amount at the end of second year. (a) ` 48,000 (b) ` 96,000
(ICSE 2010) (c) ` 1,90,000 (d) ` 1,92,000
2. Rohit borrows ` 86,000 from Arun for two years at
7. The difference between C.I. (compound interest and
5% per annum simple interest. He immediately lends
S.I. (simple interest) on a sum of ` 4,000 for 2 years
out this money to Akshay at 5% compound interest

d
at 5% p.a. payable yearly is
compounded annually for the same period. Calculate

ite
Rohit’s profit in the transaction at the end of two years. (a) ` 20 (b) ` 10 ((cc)) ` 50
(c) (d) ` 60
(ICSE 2010)

m
8. If the difference between simple interest and
3. Mr. Kumar borrowed ` 15,000 for 2 years. The rate compound interest on a certain sum of money for

Li
of interest for the two successive years are 8% and 3 years at 10% per annum is ` 31, the sum is
10% respectively. If he repays ` 6,200 at the end of (a) ` 500 ((bb)) ` 750
(b) (c) ` 1000 (d) ` 1250

ny
the first year, find the outstanding amount at the end
9. On what sum of money will compound interest for
of the second year. (ICSE 2011)
2 years at 5% per annum amount to ` 164?
4. In what period of time will ` 12,000 yield ` 3,972 as
compound interest at 10% per annum, if compounded pa
((aa)) ` 1600
(a) (b) ` 1500 (c) ` 1400 (d) ` 1700
om
on an yearly basis? (ICSE 2011) 10. A sum of money becomes eight times in 3 years. If
the rate is compounded annually, in how much time
5. On what sum of money will the difference between will the same amount at the same compound rate
C

the compound interest and simple interest for 2 years become sixteen times?
be equal to ` 25 if the rate of interest charged for both
d

(a) 6 years (b) 4 years (c) 8 years (d) 5 years


is 5%?
An

ANsWErs
d

1. (i) 12% p.a. (ii) ` 7,526.40 2. ` 215 3. ` 11,000 4. 3 years 5. ` 10,000


n

5 5
r  r 
ha

 
6. (d) [Hint: 24000 = 12000 11+
+  ⇒ 1 +  =2
 100   100 
C

20 5 4 
 r   r  
S

∴ After 20 years, A = 12000 1 + = 12000 1 + 12000 × 24 


   = 
 100   100   
©

7. (b) 8. (c) 9. (a) 10. (b)


UNIT 3 ALGEBRA

3
CHAPTER EXPANSIONS

d
ite
RECALL OF CONCEPTS LEARNT IN EARLIER CLASSES

m
I. Products of two binomials
(x + a) (x + b) = x2 + (a + b) x + ab

Li
Thus, (x + 3) (x + 4) = x2 + (3 + 4) x + 3 × 4 = x2 + 7x + 12

ny
(x − 3) (x − 4) x + (−3 − 4) x + (−3) (−4) = x − 7xx + 12
2 2
=
(x − 3) (x + 4) x + (−3 + 4) x + (−3) (+4) = x + x − 12
2 2
=
(x + 3) (x − 4)
pa
x + (+3 − 4) x + (+3) (−4) = x − x − 12
2 2
=
In general,
om
(x + a) (x + b) = x + (algebraic sum of second terms) x + product of second terms.
2

Note. ‘ax’’ is sometimes called the inner ‘product’ and ‘bx


‘‘bx’
bx’’ the outer product’.
C

Thus, (x − a) (x − b) = x2 + (− a − b) x + ((− − a) ((− b)


d

2
= x − (a + b) x + ab
An

(x + a) (x − b) = x + (a − bb)) x + ((a) (−b)


2
2
= x + (a − b) x − ab
(x − a) (x + b) = x + (− −aa + b) x + (−a) (b)
2
d

((−a
= x + (b (b − a) x − ab
2
n
ha

Corollary. If a and b are equal, then


(x + a) (x + a) = x + (a + a)x + (a) (a)
2
2 2
C

= x + 2ax + a
(x − a) (x − a) = x + (−a − a) x + (−a) (−a)
2
S

= x − 2ax + a
2 2

(x + a) (x − a) = x2 + (a − a) x + (a) (−a)
©

2 2
= x −a.
Now suppose that you have to find product of any two binomials ax + by and cx + dy. Then,
(ax + by) (cx + dy) = (ax + by) cx + (ax + by) dy
= acx2 + bcxy + adxy + bdy2
= acx2 + (bc + ad) xy + bdy2
Method : Step 1. Multiply the first terms of the binomials.
Step 2. Find the inner and outer products and take their algebraic sum.
Step 3. Multiply the last terms of the binomials.
Last step. The required product is the algebraic sum of the products in steps 1 to 3.
Ch 3–2 ICSE MATHEMATICS–IX

Ex. 1. Multiply : (2x + 6) (3x + 4). Step 2


Sol. Step 1. (2x) (3x) = 6x2 Step 2
Step 2. (+6) (3x) + 4 (2x) = + 18x + 8x = + 26x
Step 3. (+6) (+4) = + 24 (2x + 6) (3x + 4)
Last step. The product is 6x2 + 26x + 24.
Ex. 2. Find (7x − 3) (3x − 3) by inspection. Step 1
Step 3
Sol. (7x) (3x) + [(−3) (3x) + (−3) (7x)] + (−3) (−3)
= 21x2 + (−9x − 21x) + 9 = 21x2 − 30x + 9.
II. Square of a binomial
A. The formula for (a + b)2

d
ite
(a + b)2 = (a + b) (a + b) = (a) (a) + (ab + ab) + (b) (b)
2 2
= a + 2ab + b (Q ab = ba)

m
Therefore,
(a + b) = a + 2ab + b
2 2 2

Li
Method : Step 1. Square the first term of the binomial.

ny
Step 2. Double the product of two terms, + a and + b.
Step 3. Square the second term of the binomial.
B. The formula for (a − b)
pa
2

(a − b)2 = (a − b) (a − b) = (a) (a) + (−ab − ab) + (−b)


bb)) (−−bb))
((−b)
om
= a − 2ab + b
2 2

Therefore,
C

(a − b) = a − 2ab + b
2 2 2
d

Method : Step 1. Square the first term of the binomial.


An

Step 2. Double the product of two terms, + a and − b.


Step 3. Square the second term of the binomial.
2
d

Note that this formula is of the same form as that of the expansion of ((a + b) except that here you take minus
twice the product ‘ab’’ instead of plus twice the product ‘‘ab’.
n
ha

Other forms of the above formulas


2ab ⇒ a2 + b2 + 2ab = (a + b)2 ⇒ a2 + b2 = (a + b)2 − 2ab
2 2 2
(i) (a + b) = a + b + 2ab
(ii) (a − b) = a + b − 22abab ⇒ a2 + b2 − 2ab = (a − b)2 ⇒ a2 + b2 = (a − b)2 + 2ab
2 2 2
C

2 2 2 2 2 2
(iii) (a + b)) + ((aa – bb) = (a + b + 2ab) + (a + b – 2ab)
S

(a + b) + (a – b) = 2(a + b )
2 2 2 2
©

(iv) (a + b)2 – (a – b)2 = (a2 + b2 + 2ab) – (a2 + b2 – 2ab)

(a + b)2 – (a – b)2 = 4ab

(v) (a + b) (a – b) = (a)(a) + (ab – ba) + (b)(– b) = a2 – b2 (Q ab = ba)

Hence, (a + b) (a – b) = a2 – b2
(vi) It follows that
2 2
 1 1 1 1
 a +  = (a) + 2 ⋅ a ⋅ +   = a2 + 2 + 2 .
2
 a a a a
EXpAnSIonS Ch 3–3

2 2
 1 1 1 1
 a −  = (a) – 2 ⋅ a ⋅ +   = a2 – 2 + 2 .
2
 a a a a

Illustrations

1. (2x + 3y)2 = (2x)2 + 2 ⋅ (2x) ⋅ (3y) + (3y)2 = 4x2 + 12xy + 9y2. (Using (a + b)2 = a2 + 2ab + b2)
2 2 2
 4x y   4x   4 x  y   y 
2.  +  =   + 2 ⋅    +  
 5 3  5   5  3   3 
16 2 8 1
x + xy + y 2 .

d
=
25 15 9

ite
3. (3x – 4y)2 = (3x)2 – 2 . (3x)(4y) + (4y)2 (a – b)2 = a2 – 2ab + b2)
(Using (a

m
2 2
= 9x – 24xy + 16y .
2 2 2
= (a2)2 – 2(a2)(b2) + (b2)2

Li
4. (a – b )
= a4 – 2a2b2 + b4.

ny
5. (x + 3) (x − 3) = (x) − (3) = x − 9.
2 2 2

6. (5x + 7y) (5x − 7y) = (5x) − (7y) = 25x − 49y .


2 2 2 2

7. (a − b − c) (a − b + c) = [(a − b) − c] [(a − b) + c] pa
om
= (a − b) − (c) = a − 22ab
ab + b2 − c2.
2 2 2

8. (1 + x) (1 + x ) (1 − x) = [(1 + x) (1 − x)]
2
)] (1 + x2)
C

= (1 − x ) (1 + x ) = (1 − x ) (1 + x )
2 2 2 2 2
d

(x2)2 = 1 − x4.
= (1) − (x
2
An

III. Perfect square


Since a2 + 2ab + b2 is equal to (a
(a + bb))2, therefore, it is a perfect square. Thus,
n d

A given polynomial will be perfect square if


ha

(i) it has three terms;


(ii) two of the three terms are perfect squares ;
C

(iii) the third term is equal to twice the product of the square roots of the two perfect square terms.
S

For instance,
©

1. 4x2 + 20xy 25y2 is a perfect square because


20xy + 25
(i) it has three terms, (ii) two terms, namely, 4x2 and 25y2 are perfect squares as 4x2 = (2x)2 and 25y2 = (5y)2,
(iii) the third term 20xy = 2(2x) (5y), i.e., it is equal to twice the product of the square roots of 4x2 and
25y2.
(ii) 16 – 8x + x2 = (4)2 − 2 × (4) × (x) + (x)2
This also satisfies the three conditions. So, it is a perfect square because it can be written as (4 − x)2.
(iii) The expression a2 – 12a – 36 is not a perfect square because the third term (−36) is negative.
(iv) y2 − 8y + 9 = (y)2 − 8y + (3)2
Since the middle term (−8y) is not equal to twice the product of y and 3, therefore this expression is not a perfect
square.
Ch 3–4 ICSE MATHEMATICS–IX

IV. Formula for square of a trinomial, i.e., (a + b + c)2


Let a+b = x, Then (a + b + c)2 = (x + c)2
(x + c)2 = x2 + 2xc + c2
Putting x =a+b
(a + b + c)2 = (a + b)2 + 2 (a + b) c + c2
= a2 + 2ab + b2 + 2ac + 2bc + c2
= a2 + b2 + c2 + 2 (ab + bc + ca)
∴ (a + b + c)2 = a2 + b2 + c2 + 2(ab + bc + ca)
a2 + b2 + c2 = (a + b + c)2 – 2(ab + bc + ca)
Note: It follows from the above chat

d
(a ± b ± c)2 = a2 + (± b)2 + (± c)2 + 2(a) (± b) + 2 (± b) (± c) + 2 (a) (± c)

ite
= a2 + b2 + c2 + 2 (a) (± b) + 2 (± b) (± c) + 2 (a) (± c)

m
Illustrations:
(i) (2x + 4y + z)2 = (2x)2 + (4y)2 + (z)2 + 2(2x)(4y) + 2(4y)(z) + 2(z)(2x) )(2xx))
)(2

Li
= 4x + 16y + z + 16xy + 8yz + 4zx.
2 2 2

(ii) (a + b – 2c) = (a)2 + (b)2 + (–2c)2 + 2(a)(b) + 2(b)(–2c)) + 2(–2c


2
2(–2
2(–2c)(a)
c)(
)(aa

ny
= a + b + 4c + 2ab – 4bc – 4ca.
2 2 2

(iii) (–3a + b – c) = (–3a)2 + (b)2 + (–c)2 + 2(–3a)(b)) + 2(b


2

pa
2(
2(b)(–c)
b)(–
)(–cc)) + 2(–
2(–c)(–3a)
= 9a + b + c – 6ab – 2bc + 6ca.
2 2 2
ca.
om
(iv) (a2 + b2 + c2)2 = (a2)2 + (b2)2 + (c2)2 + 2(a2)(
)(bb2) + 2(b
2(b2)(
)(c2) + 2(c2)(a2)
= a4 + b4 + c4 + 2a2b2 + 2bb2c2 + 2c
2c2a2.
C

V. Formula for the cube of a binomial


A. The Formula for (a + b)3.
d

Aid to Memory
An

(a + b) = a + 3aa b + 3ab3ab2 + b3
3 3 2

(a + b) = (1st term3 + 3 × 1st term2 ×


3
(a + b)3 = a3 + b3 + 3ab
3ab (a + b)
↑ ↑ 2nd term + 3 × 1st term × 2nd
a + b = ((aa + bb)) − 3ab (a + b)
3 3 3
d

1st 2nd term2 + 2nd term3)


term term
n

B. The formula for (a – bb))3.


ha

a − bb)) = a − 3a b + 3ab − b
3 3 2 2 3
(a Aid to Memory
((aa − b)
b)3 = a3 − b3 − 3ab (a − b)
C

(a – b)3 = (1st term3 – 3 × 1st term2 ×


a − b = (a − b) + 3ab (a − b)
3 3 3
↑ ↑
S

2nd term + 3 × 1st term × 2nd


1 1st 2nd 2 3
Cor. (i)) Putting a = x and b = , we have term term term – 2nd term )
x
©

3
 1 1 1 1
x+  = x3 + 3
+ 3. x .  x + 
 x x x x
3
 1 1  1
i.e, x+  = x +
3
3
+ 3x + .
 x x  x
Also it follows from the above that
3
1  1  1
x3 + =  x +  − 3 x + .
x3  x  x
3
 1 3 1 1 1
(ii) x−  = x − 3
− 3.x.  x − 
 x x x  x
EXpAnSIonS Ch 3–5

3
 1 3 1  1
⇒ x−  = x − −3x − .
3
 x x  x
3
1  1  1
⇒ x3 − = x−  + 3 x − .
x3  x  x
Illustrations:
3 3 2 2 3
(i) (2x + 3y) = (2x) + 3(2x) (3y) + 3(2x)(3y) + (3y)
= 8x3 + 3(4x2)(3y) + 3(2x)(9y2) + 27y3
= 8x3 + 36x2y + 54xy2 + 27y3.
3 2 3
3 1

d
 1 3 2 1 1 1
(ii)  x +  = x + 3x ⋅ + 3x ⋅   +   = x3 + 3x + + 3 .

ite
 x x  x  x x x
3 3 2 2 3
(iii) (4x – 5y) = (4x) – 3(4x) (5y) + 3(4x)(5y) – (5y)

m
= 64x3 – 3 × 16x2 × 5y + 3 × 4x × 25y2 – 125y3

Li
= 64x3 – 240x2y + 300xy2 – 125y3.
3 2 3
 1  2 1   1   1 

ny
3
(iv)  2 x −  = (2 x) − 3(2 x)   + 3(2 x)   −  
 3y   3y   3y   3y 
3 2  1   1 
= 8 x − 3(4 x )   + 3(2 x)  2  −
 3y 
1
 9 y  27 y
3 pa
om
3 4 x2 2 x 1
= 8x − + 2 − .
y 3y 27 y 3
C

ExErCISE
ExE
Ex Er
rCISE
CISE 3 (a)
d
An

1. Write down the products for each of the 5. Find the products:
following: (i) (x + y) (x – y) (x2 + y2)
(i) (x + 4) (x + 2) (ii)) (4
(4aa –5) (5
(5a + 6) (ii) (a + b ) (a + b ) (a + b) (a – b)
2 2 4 4
d

2 2
(iii) (xy + 6) (xy – 5) ((iv
(iv) (7x – 55y) (x – 3y)
iv)) (7x
n

6. State which of the following expressions is a


2. Write down the squares of the following
ha

perfect square.
expressions 2
(i) x + 8x + 16 (ii) y2 + 3y + 9
(i) 3x + 5y (ii)
((ii
ii 5y – 2z
C

(iii) 4m2 – 4m + 1
1 1
(iii) 5 p − (iv) (5x + 3y + z)
2
(iv) 4x2 – 2 + (v) m2 + 6m + 4
S

44qq 4x 2

2
7. If 4x – 12x + k is a perfect square, find the
©

2
 1 
m – 5n (vi)  2 x − p + 3q 
2
(v) (–3m 5 + 2p)
3 numerical value of k. (ICSE)
 
8. What term should be added to each of the
3. Simplify: (2x – p + c)2 – (2x + p – c)2 following expressions to make it a perfect
4. Write down the following products: square?
2
(i) 4a + 28a (ii) 36a2 + 49b2
1 1 
(i) (3b + 7) (3b – 7) (ii)  − 5 x   + 5 x  (iii) 4a2 + 81 (iv) 9a2 + 2ab + b2
3 3 
(v) 49a4 + 50a2b2 + 16b4
3 3 9. Write down the expansions of the following:
(iii) (x – 3) (x + 3)
3
 4 1  4 1  (i) (a + 1) (ii) (3x – 2y)3
(iv)  a −   a +  3 3
 5y   5y   1  a b
(iv)  2 x −  (v)  + 
2 3
(iii) (x + y)
 3x  5 2
Ch 3–6 ICSE MATHEMATICS–IX

answers
2 2
1. (i) x + 6x + 8 (ii) 20a – a – 30 (iii) x2 y2 + xy – 30 (iv) 7x4 – 26x2 y + 15y2
2 5p 1
2. (i) 9x2 + 30xy + 25y2 (ii) 25y2 – 20yz + 4z2 (iii) 25 p − +
2q 16q 2
2 2 2
(iv) 25x + 9y + z + 30xy + 6yz + 10xz (v) 9m2 + 25n2 + 4p2 + 30mn – 20np – 12mp
2 1 2 2 4
(vi) 4 x + p + 9q − xp − 2 pq + 12 xq
9 3
1 1
3. –8xp + 8xc
2
4. (i) 9b – 49 (ii) − 25 x 2 6
(iii) x – 9
8
(iv)) a −
9 25 y 2

d
4 4
5. (i) x – y (ii) a8 – b8 6. (i), (iii) and (iv) 7. k = 9

ite
8. (i) 49 (ii) 84ab (iii) 36a (iv) 4ab (v) 6a2 b2
3 2
(ii) 27x – 54x y + 36xy – 8y3
3 2 2
(iii) x6 + 33xx4y + 3x
3x2y2 + y3

m
9. (i) a + 3a + 3a + 1

2 1 a3 3a 2b 3ab 2 b3

Li
3
(iv) 8 x − 4 x + − (v) + + +
3 x 27 x3 125 50 20 8

ny
More Examples based on these expansions

Ex. 1. Evaluate each of the following using identities:


pa
om
(i) 105 × 95 (ii) 204 × 204 (iii) 96 × 96 (iv) 165 × 165 – 135 × 135
2 2
Sol. (i) 105 × 95 = (100 + 5)(100 – 5) Q (a + b) (a – b) = a – b
C

= (100)2– (5)2 = 10000 – 25 = 9975.


2 2
(ii) 204 × 204 = (204) = (200 + 4)
d

2 2
An

= (200) + 2(200) (4) + (4)


2 2 2
= 40000 + 1600 + 16 = 41616. Q (a + b) = a + 2ab + b
(iii) 96 × 96 = (96)2 = (100 – 4)2
d

2 2 2 2 2
Q (a – b) = a – 2ab + b
n

= (100) – 2.(100)(4) + (4)


ha

= 10000 – 800 + 16 = 10016 – 800 = 9216.


2 2
(iv)165 × 165 – 135 × 135 = (165) – (135)
C

= (165 + 135)(165 – 135)


2 2
Q (a – b ) = (a + b) (a – b)
S

= 300 × 30 = 9000.
2 2
Ex. 2. Find the value of (i) a + b when a + b = 7 and ab = 12.
©

1 1 1
(ii) x + and x + if x + =
2 4
2 4
4
x x x
Sol. (i) a + b = (a + b) − 2ab
2 2 2

= 7 − 2 × 12 = 49 − 24 = 25.
2

2 2
1  1 1  1
=  x +  − 2 × x × = x +  − 2
2
(ii) x +
x2  x x  x
= 4 − 2 = 16 − 2 = 14.
2

2
4 1  2 1  2 1
Now, x + =  x + 2  − 2 × x × 2 = 14 − 2 = 196 − 2 = 194.
2
x4  x  x
EXpAnSIonS Ch 3–7

1
Ex. 3. If a2 + = 7, find the values of
a2
1 2 1 1
( i ) (a + ) (ii) (a – ) (iii) (a2 – )
a a a2
1 2 1 1
Sol. (i) (a + ) = a2 + 2 + 2 = 7 + 2 = 9 ⇒ a + = ± 3.
a a a
1 2 1 1
(ii) (a – ) = a2 + 2 – 2 = 7 – 2 = 5 ⇒ a – = ± 5.
a a a
1 1 1
(iii) (a – 2 ) = (a – ) (a + ) = (± 3)( ± 5 ) = ± 3 5 .
2

d
a a a

ite
2 2 2
Ex. 4. Find the value of a + b + c if a + b + c = 13 and ab + bc + ca = 27.
(a + b + c)2 = a2 + b2 + c2 + 2(ab + bc + ca)

m
Sol.
Substituting the values, we have

Li
(13)2 = a2 + b2 + c2 + 2 × 27
⇒ a2 + b2 + c2 = (13)2 – 54 = 169 – 54 = 115

ny
⇒ a2 + b2 + c2 = 115.
Ex. 5. If a2 + b2 + c2 = 5 and ab + bc + ca = 10, find the value of a + b + c.
Sol. (a + b + c)2 = a2 + b2 + c2 + 2(ab + bc + ca) pa
om
= 5 + 2 × 10 = 5 + 20 = 25
∴ a + b + c = ± 25 = ± 5.
C

3 2 2 3
Ex. 6. Find the value of 27a + 108a b + 144ab + 64b if a = 2, b = 3.
Sol. The given expression
d

3 2
= (3a) + 3 (3a) (4b) + 3(3a) aa)) (4b
(4 b))2 + (4
(4b) (4b)3
An

= (3a + 4b)3 = (3 × 2 + 4 × 3)3 = (6 + 12)3 = 183 = 5832.


Ex. 7. Find the value of
d

3 3
(i) x + y if x + y = 6 and xy = 5;
n

3 3
(ii) 27x + 8y if 3x + 2y = 14 and xy = 8.
ha

Sol. (i) x + y = (x + yy)) − 33xy


3 3 3
xy (x
(x + y)
= 6 − 3 × 5 × 6 = 216 − 90 = 126.
3
C

(2y)3 = (3x + 2y)3 − 3 × 3x × 2y (3x + 2y)


3 3
(ii) 27x + 88yy = (3 x 3 + (2
(3x)
(3x
S

= 14 − 18xy × (14) = 14 − 18 × 8 × 14
3 3

= 2744 − 2016 = 728.


©

Ex. 8. Find the value of p3 − q3 if p − q = − 8, pq = −12.


Sol. p3 − q3 = (p − q)3 + 3pq (p − q)
= (−8)3 + 3(−12) (−8) = −512 + 288 = −224.
1 1 1
Ex. 9. Find the value x3 + 3
if (i) x + = −1 (ii) x − = 5 .
x x x
3
 1 3 1  1
Sol. (i)  x +  = x + 3 + 3  x + 
 x x  x
3
3 1  1  1
⇒ x + 3 = x +  − 3x + 
x  x  x
= (−1) − 3(−1) = −1 + 3 = 2.
3
Ch 3–8 ICSE MATHEMATICS–IX

1
(ii) First, we find the value of x + .
x
Using the formula, (a + b) = (a − b) + 4ab, we have
2 2

2 2
 1  1 1
⇒ x+  = x−  +4 = ( 5 )2 +4=5+4=9 ⇒ x+ =±3
 x  x x

3
1  1  1
Now, x3 + = x +  − 3x + 
x3  x  x

= (3)3 − 3(3) = 27 − 9 = 18, when x = 3

d
1
x3 + = (−3) − 3(−3) = −27 + 9 = −18, when x = −3.
3

ite
and 3
x
1 1 1

m
Ex. 10. Find the value of x3 − 3
if (i) x − = 6, (ii) x + = 29 .
x x x

Li
3
1  1  1
3
Sol. (i) x – = x −  + 3x − 
x3

ny
 x  x
= (6)3 + 3(6) = 216 + 18 = 234.

(ii) First, we find x −


1
x
. pa
om
2 2
 1  1 1
 x −  =  x +  − 4.x.
C

 x  x x
2
 1
( 29 )2 − 4 = 29 − 4 = 25.
d

= x+  −4=
 x
An

1
∴ x− =±5
x
d

3
1  1  1
n

Now, x3 − = x −  + 3x − 
x3 x x
ha

 
= (5)3 + 3 × 5 = 125 + 15 = 140, when x = 5
C

1
and x3 − 3
= (−5) + 3 (−5) = −125 − 15 = −140, when x = – 5
3
x
S

Q − ×+ = −
1 2 1 −×− = +
) = 3, show that x3 + 3 = 0.
©

Ex. 11. If (x +
x x
2
 1 1
Sol. x+  = 3 ⇒ x+ = ± 3
 x x
3
 1 3 1  1
Now,  x +  = x + 3 + 3⋅ x + 
 x  x  x
3
1  1  1
⇒ x3 + =  x +  – 3⋅ x + 
x3  x  x
3
= (± 3) − 3 ⋅ (± 3)
= ± 3 3  3 3 = (3 3 − 3 3) o r( − 3 3+ 3 3)
=0
EXpAnSIonS Ch 3–9

Ex. 12. If a + b + c = 0, show that a3 + b3 + c3 = 3abc.


Sol. Given : a+b+c = 0
⇒ a + b = –c
⇒ (a + b)3 = (–c)3 [Cubing both sides]
⇒ 3 3
a + b + 3ab (a + b) = –c 3

⇒ a3 + b3 + 3ab (–c) = –c3 [Since a + b = –c]


⇒ 3 3 3
a + b + c – 3abc = 0
⇒ a3 + b3 + c3 = 3abc.
Note. The above is an important result and should be memorised.

d
Ex. 13. Find the value of (55)3 − (25)3 − (30)3.

ite
Sol. Let a = 55, b = −25, c = −30

m
Then, a + b + c = 55 − 25 − 30 = 0
⇒ a3 + b3 + c3 = 3abc

Li
⇒ (55)3 + (−25)3 + (–30)3 = 3(55) (−25) (−30) = 123750.

ny
( x + y )2 ( y + z )2 ( z + x )2
Ex. 14. If x3 + y3 + z3 = 3xyz and x + y + z = 0, find the value of + + .
xy yz zx
Sol. x + y + z = 0 ⇒
2
( x + y)
x + y = –z, y + z = –x and z + x = ––yy
( y + z) 2 ( z + x) 2 (− z) 2 (− x) 2 (− y) 2 paz 2 x2 y 2
om
⇒ + + = + + = + +
xy yz zx xy yz yx xy yz zx
z 3 + x3 + y 3 3 xyz
C

= = = 3. 3 3 3
[Given : x + y + z = 3xyz]
xyz xyz
d

ExE
Ex E
ExErCISE 3 (b)
An

1. Find the value of: 3. (i) 8x3 + 84x2y + 294xy2 + 343y3 if x = 1, y = 2


2 2
(i) a + b when a + b = 9, ab = 20 [Hint: Given exp. (2x)3 + 3 (2x)2 (7y) +
d

(ii) p + q2 if p – q = 6 and p + q = 14
2
2 3 3
n

3(2x) (7y) + (7y) = (2x + 7y)


(iii) mn if m + n = 8 and m – n = 2 3 2 2 3
ha

(ii) 27x – 27x y + 9xy – y if x = 2, y = 1


2 1 4 1 1
(iv) x + 2 and x + 4 if x + = 3 4. (i) a3 + b3 if a + b = 3 and ab = 2
x x x
1 1
C

3 
1 1 1 (ii) x + 3 if  x +  = 3
2
(v) x − , and x − 2 if x + =5 x  x
S

x x x
3 1 2 1
2. (i) a2 + b2 + c2 if a + b + c = 17 and (iii) x − 3 if x + 2 = 18
x x
©

ab + bc + ca = 30
3 1 2 1 3
(ii) ab + bc + ca if a + b + c = 15 and (iv) x + 3
if x + 2
8
= (ICSE)
125 x 25 x 5
a2 + b2 + c2 = 77
(iii) a + b + c if a2 + b2 + c2 = 50 and 5. Evaluate: (i) 102 × 98 (ii) 10032 – 9972
ab + bc + ca = 47 (iii) (10)3 – (5)3 – (5)3

anSwErS
1. (i) 41 (ii) 116 (iii) 15 (iv) 7; 47 (v) ± 1; ± 5
2. (i) 229 (ii) 74 (iii) ± 12 3. (i) 4096 (ii) 125
1
4. (i) 9 (ii) 18 (iii) ± 76 (iv) ± 25
5
5. (i) 9996 (ii) 12000 (iii) 750
Ch 3–10 ICSE MATHEMATICS–IX

ChaPtEr tESt
1. The coefficient of x in the product (2 – 3x) (5 – 2x) is
(a) 19 (b) – 19 (c) 15 (d) 6
2. If 3x + kx – 8 = (3x – 2) (x + 4) for all x, then the value of k is:
4 2 2 2

(a) – 2 (b) 12 (c) 10 (d) – 8


 1
3. The coefficient of x2 in (3x + x3)  x +  is
 x
(a) 3 (b) 1 (c) 4 (d) 2

d
3 3
4. If a = 3 + b, prove that a – b – 9ab = 27.

ite
(a 2 − b 2 )3 + (b 2 − c 2 )3 + (c 2 − a 2 )3
5. Simplify:
(a − b)3 + (b − c)3 + (c − a)3

m
1 1

Li
3
6. If a + = 0, then the value of (a + 2) + is
(a + 2) (a + 2)3

ny
(a) 6 (b) 4 (c) 3 (d) 2

pa
1  1 
7. If a + +2=0, then the value of  a37 − 100  is
a  a 
om
(a) 0 (b) –2 (c) 1 (d)
((d
d) 2
d)
2 2 2
8. If (a – 1) + (b + 2) + (c + 1) = 0 then the value of 2a
2a – 33b + 7c is
C

(a) 12 (b) 3 (c) – 11 (d) 1


9. If ax + by = 3, bx – ay = 4 and x + y = 1, then the value of a2 + b2 is
2 2
d
An

(a) –1 (b) –25 (c)


((c
c)) 1 (d) 25

p q
10. If p + q = 10 and pq = 5, then the numerical value of + will be:
d

q p
n

(a) 22 (b) 18 (c) 16 (d) 20


ha

anSwErS
C

1. (b) 2. (c) 3. (c)


S

(b + cc)) (c
5. (a + b)) (b (c + aa) [Hint: Use the result if a + b + c = 0 then a3 + b3 + c3 = 3 abc]
©

1
6. (d) [Hint: a + 0 ⇒
= 2
a + 2a + 1 = 0 ⇒ (a + 1)2 = 0 ⇒ a = – 1]
a+2
2 2 2
7. (b) 8. (d) [Hint: If a + b + c = 0, then a = b = c = 0]
9. (d) 10. (b)
4
CHAPTER FACTORISATION

d
ite
Factorising polynomials
If x − 2 is multiplied by 4, the answer is 4x − 8. Thus, 4 and (x − 2) are the factors of 4x4x − 8. Since both 4x and

m
−8 can be divided by 4, therefore 4 is the common factor of 4x − 8.

Li
Type 1. When factorising, common factors are always taken out first.
Ex. 1. Factorise:

ny
3 2 4 3 2 2 3
(a) 6p + 8q (b) 7xy + 14yz (c) 28x − 70x (d) 20x y − 12x y + 28x y .
Sol. (a) 6p + 8q = 2(3p + 4q) ;
(b) 7xy + 14yz = 7y(x + 2z) ;
(c) 28x − 70x = 14x (2x − 5) ;
3 2 2 pa (a) Take out the common factor 2.
(b) Take out the common factor 7y.
om
(c) Take out the common factor 14x2.
(d) 20x y − 12x y + 28x y
4 3 2 2 3
(d) Take out the common factor 4x2y.
= 4x2y (5x2 − 3xy + 7y2).
C

Ex. 2. Factorise : x (a + b) + y (a + b).


Sol. The common factor is (a + b)
d

∴ x(a + b) + y(a + b) = (a + b) (x + yy). ).


An

3 2
Ex. 3. Factorise : 12 (a − b) + 18 (a − b) .
Sol. The factor common to both terms is 6( 6(a − b) .
2
d

∴ 12 (a − b) + 18 (a − b) = 6 (a (a − bb)2 [2 (a − b) + 3] = 6 (a − b)2 (2a − 2b + 3).


3 2
n
ha

EXERCISE 4 (a)
Factorise :
C

1. 8x + 16y 2. ax − 2ay 3. 25xy − 50y 4. abc − ac


5. −12l m − 11 m 6. a b + ab 7. 24x3y − 30x2y 8. −9x3 − 12x2 + 3x
2 2 2
S

9. 25a bc − 35 10. − x − y − z 11. x (x − y) + (x − y)


2 3
ab2c3 + 45a3bc2
35ab
©

12. (x + y) − (x 13. a(x − y) −b (y − x) 14. (a − b)4 + (a − b)3


2
(x + y)
15. (x − y) + (y − x) [Hint. Given exp = (x − y) − (x − y) ]
6 5 6 5

ANSWERS
1. 8(x + 2y) 2. a(x − 2y) 3. 25y(x − 2) 4. ac(b − 1)
5. 11m (–11 − m) 6. ab(a + b) 7. 6x y(4x − 5)
2
8. −3x(3x2 + 4x − 1)
9. 5abc (5ac − 7bc + 9a ) 10. −1(x + y + z) 11. (x − y) (x + 1) 12. (x + y) (1 − x − y)
2 2

13. (x − y) (a + b) 14. (a − b) (a − b + 1) 15. (x − y)5 (x − y − 1).


3
Ch 4–2 ICSE MATHEMATICS–IX

Type 2. Factors found by grouping terms.


Consider the expression ax + ay + bx + by.
It appears that the expression as it stands, has no common factor to all the terms, but if we notice carefully we
find that the first two terms have a as a common factor and the last two terms have b as a common factor. Enclosing
the first two terms in one bracket and last two terms in another, we have,
ax + ay + bx + by = (ax + ay) + (bx + by) = a(x + y) + b (x + y)
= (x + y) (a + b). (x + y) is the common factor
2
Ex. 1. Factorise : a + bc + ab + ac.
Sol. a2 + bc + ab + ac
= (a2 + ab) + (bc + ac) = a(a + b) + c(b + a)

d
= a(a + b) + c (a + b) = (a + b) (a + c). ((a
a + bb)) is the common factor

ite
Ex. 2. Factorise : 3mn + 2pn + 3mq + 2pq.
Sol. 3mn + 2pn + 3mq + 2pq

m
Rearranging the whole expression we have

Li
3mn + 3mq + 2pn + 2pq
= 3m (n + q) + 2p (n + q) ...(i)

ny
= (n + q) (3m + 2p) ...(ii)
In (i), 3m is a common factor of the first pair, and 2p of the second pair. In ((ii) (n + q) is a common factor.
Ex. 3. Factorise: ax – ay – 3az + 2bx – 2by – 6bz.
Sol. Rearranging the whole expression we have pa
om
ax + 2bx – ay – 2by – 3az – 6bz
= x(a +2b) – y(a + 2b) – 3z(a + 2b
22b)
b))
C

= (a + 2b) (x – y – 3z) Taking (a + 2b) common


2 2
Ex. 4. Factorise: (ax + by) + (bx – ay) .
d

Sol. (ax + by)2 + (bx – ay)2


An

= a2x2 + 2abxy + b2y2 + b2x2 – 2abxy + a2y2


= a2x2 + b2y2 + b2x2 + a2y2
d

Rearranging the terms we have,


n

= a2x2 + b2x2 + b2y2 + a2y2


ha

= x2((aa2 + b2) + y2(b2 + a2) Q a2 + b2 = b2 + a2


= ((aa2 + b2) (x2 + y2) Taking (a2 + b2) common
C

ExErCisE 4 (b)
S

Factorise the following.


©

1. bx + 2bb + cx + 2c 2.
3 2
y + 2y + 3y + 6
3. xa + 3b + xb + 3a 4. 8xy + 5zy – 8xt – 5zt
5. 8kl + 12ml – 12mn – 8kn 6.
2
32(x + y) – 2x – 2y
7. x – x + ax + x – a –1 8.
3 2
ab(c2 + 1) + c(a2 + b2)
9. a – a + xa + a – x – 1 10.
3 2
6a3b + 3a2b2 – 2a2b – ab2
11. a3 + ab(1 – 2a) – 2b2 12. (p2 + 1)q – p2 – q2

AnswErs
1. (b + c) (x + 2) 2. (y + 3) (y + 2) 3. (a + b) (x + 3) 4. (y – t) (8x + 5z)
2

5. 4(l – n) (2k + 3m) 6. 2(x + y) (16x + 16y – 1) 7. (x – 1) (x + a + 1) 8. (ac + b) (a + bc)


2

9. (a – 1)(a + x + 1) 10. ab(3a – 1)(2a + b) 11. (a2 + b)(a – 2b) 12. (p2 – q) (q – 1)
2
FACTorISATIon Ch 4–3

Factorising a perfect square trinomial


A perfect square trinomial is the square of a binomial. Thus, a2 + 2ab + b2 and a2 − 2ab + b2 are perfect square
trinomials because they are squares of a + b and a − b respectively.
The characteristics of a perfect square trinomial may be summarised as follows :
(1) The first and third terms are perfect squares.
(2) The absolute value (i.e., plus or minus) of the middle term is twice the product of the positive square roots of
the first and third terms.
4 2
Ex. 1. Factorise : 4x + 12x + 9.
Sol. 4x + 12x + 9 = (2x ) + 2(2x ) (3) + 3
4 2 2 2 2 2

= (2x2 + 3)2 = (2x2 + 3) (2x2 + 3)

d
1
Ex. 2. Factorise : a − 2 + .
2

ite
a2
2
1 1 1
a2 − 2 +

m
Sol. = (a) − 2(a )   +  
2
a2 a a

Li
2
 1  1 1
=  a −  =  a −   a − .
 a  a a

ny
ExErCisE 4 (c)
Factorise :
1. x + 4x + 4
2
2. x2 + 6x + 9
pa
3. x2 − 10 4. 4x2 − 4x + 1
om
10x + 25
1 2 5p 1
5. 1 − 8x + 16x2 6. 49x4 + 168x2y2 + 144
144yy4 7. x 2 + x + 8. 25 p + +
4 2q 16q 2
C

AnswE
AnswErs
d

1. (x + 2) 2. (x + 3)2 3. (x
(x − 5)2 4. (2x − 1)2 5. (1 − 4x)2
2
An

2 2
 1  1 
6. (7x2 + 12y2)2 7.  x +  8.  5 p + 
 2  4q 
n d

Factorising the difference of two squares


ha

You have come across the products of the following form.


(a + b) (a a − bb)) = a2 − b2
C

Thus, (3x − yy)) = 9x2 − y2,


(3x + yy)) (3
(5x − 4y) (5x + 44y) = 25x2 − 16y2
yy)) (5x
S

The binomials on the right are called difference of two squares because in them two perfect squares are separated
by a minus sign.
©

We can reverse this pattern to find a pattern for factorising the difference of two squares.
Ex. 1. Factorise : 16x2 − 25y2.
Sol. 1. This is a difference of two squares so you find the
square root of the first term.
(4x) (4x)
2. Next, you find the square root of the second term.
3. Now fill in the signs. One has a plus sign, the other (4x 5y) (4x 5y)
has a minus sign. Although it does not matter which (4x + 5y) (4x − 5y)
comes first, we must have one of each. or
2 2
2 2  3  1 −
Ex. 2. Evaluate : (i) 73 − 27 (ii)  6  –  3  . (4x 5y) (4x + 5y)
 4  4
Sol. (i) 73 − 27 = (73 + 27) (73 − 27)
2 2

= 100 × 46 = 4600.
Ch 4–4 ICSE MATHEMATICS–IX

2 2
 3  1  3 1 3 1
(ii) 6  – 3  = 6 + 3  6 – 3 
 4  4  4 4 4 4
1 7
= 10 × 3 = 10 × = 35.
2 2

ExErCisE 4 (d)
Factorise the following :
1. x − 4 2. y2 − 25 3. a2 − 1 4. 9z2 − 64
2

1
5. 9x − b 6. 25 − x2y2 7. 81a2x2 – 49b2y2 8. x2 −
2 2
4
1 2 a 2 b2

d
9. −25 + b 10. − 11. 2.25a − b
2 2
12. 36
36aa − 121
8
64 9 16

ite
Evaluate the following:

m
13. 54 − 36 14. (3.2)2 − (1.8)2
2 2

Li
AnswErs

ny
1. (x + 2) (x − 2) 2. (y + 5) (y − 5) 3. (a + 1) (a − 1) 5. (3z + 8) (3z − 8)
5. (3x + b) (3x − b) 6. (5 + xy) (5 − xy) 7. (9ax + 7by
by )(9
)(9ax
ax – 7by
77by)
by


1
2
1
8.  x +   x − 
2
9.  b + 5   b – 5 
8 8  pa
a ba b
10.  +   − 
 3 4 3 4
11. (1.5 a + b) (1.5 a − b)
om
12. (6a + 11) (6a − 11) 13. 1620 14. 7
4 4

2 2
Ex. 3. Factorise : (x + y) − (a − b) .
C

Sol. Here (x + y) is one quantity and (a – b)) is another.


d


2 2
(x + y) – (a – b) = [(x + y)) + (a (a – bb)]
)] [(
[(x + y) – (a – b)]
An

= (x + y + a – b) b (x + y – a + b).
4 4
Ex. 4. Factorise : 16x – y .
Sol. 16x – y = (4x ) – (y ) = (4x
4 4 2 2 2 2
(4x2 – y2) (4x2 + y2)
d

Since the expression in the first bracket can be further factorised, therefore
n

4 4
(4x2 – y2) (4x2 + y2) = [(2x)2 – (y)2] (4x2 + y2)
ha

16xx – y = (4
2 2
= (2x – y) (2x + y) (4x + y ).
2
C

Ex. 5. Factorise : 32 – 2(x – y) .


First look for the common factor, if any.
Sol. 32 – 2(x – yy))2 = 2{16 – (x ( – y)2}
S

2 2 Here it is 2.
= 2{(4) – (x – y) }
= 2{(4 + x − y )(4 – x − y )}
©

= 2(4 + x – y) (4 – x + y).
2 2
Ex. 6. Factorise : 4(a + b) − 9(a − b) .
Sol. 4(a + b) = [2(a + b)] = (2a + 2b) . (Note this step.)
2 2 2

9(a − b) = [3(a − b)] = (3a − 3b)


2 2 2

∴ 4(a + b)2 − 9(a − b)2 = (2a + 2b)2 − (3a − 3b)2


= [(2a + 2b) + (3a − 3b)] [(2a + 2b) − (3a − 3b)]
= (5a − b) (2a + 2b − 3a + 3b) = (5a − b) (−a + 5b)
= (5a − b) (5b − a).
Sometimes a common factor of the terms of a polynomial conceals a familiar factor pattern. Therefore, the first
step in factorising a polynomial is to examine whether the given polynomial has a positive or a negative common
monomial factor and take it out if it is present.
FACTorISATIon Ch 4–5

Ex. 7. Factorise : 48ax2 − 75ay2. First take out the common factor 3a
Sol. 48ax − 75ay = 3a(16x − 25y )
2 2 2 2

= 3a[(4x) − (5y) ] = 3a(4x + 5y) (4x − 5y)


2 2

Thus the factors are 3, a, (4x + 5y), (4x − 5y).


In the next example, the first three terms of the polynomial form a perfect square. By grouping three terms, we
see that the polynomial itself can be expressed as a difference of squares.
Ex. 8. Factorise : x2 − 10x + 25 − y2.
Sol. x − 10x + 25 − y = (x − 10x + 25) − y = (x − 5) − y
2 2 2 2 2 2

= (x − 5 + y) (x − 5 − y) = (x + y − 5) (x − y − 5).
The following example also illustrates how suitable grouping helps us to express the given expression as the
difference of two squares.

d
2 2
Ex. 9. Factorise: x − y + ax + ay.

ite
Sol. x − y + ax + ay = (x − y ) + (ax + ay)
2 2 2 2

= (x + y) (x − y) + a(x + y)

m
= (x + y) [(x − y) + a] = (x + y) (x − y + a).

Li
2 2 2
Ex. 10. Factorise : 25x – 9a + 6ay – y .
Sol. 25x – 9a + 6ay – y = 25x – (9a – 6ay + y )
2 2 2 2 2 2

ny
2 2
= (5x) – (3a – y)
= (5x + 3a − y ) (5x – 3a − y )
= (5x + 3a – y)(5x – 3a + y). pa
om
2 1
Ex. 11. Factarise a + – 18.
a2
1 1
C

Sol. a +
2
2 – 18 = a2 + – 2 – 16 (Note this step)
a a2
d

2 1 1 2
= (a – 2 + ) – 16 = (a
(a – ) – (4)2
An

2
a a
1 1
= (a – (a –
+ 4) (a – 4).
a a
d

Ex. 12. Factorise the following:


n

(i) a(a − 2) − b(b − 2) (ICSE)


ha

2
(ii) x − 2y + xy − 4 (ICSE 1990)
2 2
(iii) (1 − x ) (1 − y ) + 4xy (ICSE 1990)
C

Sol. (i) a(a – 2) – b ((bb – 2) = a2 – 2a – b2 + 2b


= (a2 – b2) – (2a – 2b) = (a – b) (a + b) – 2 (a – b)
S

= (a – b) [(a + b) – 2] = (a – b) (a + b – 2).
2 2
©

(ii) x – 22yy + xy – 4 = (x – 4) + (xy – 2y)


2 2
= (x – 2 ) + y(x – 2) = (x – 2) (x + 2) + y(x – 2)
= (x – 2) (x + 2 + y) = (x – 2) (x + y + 2).
2 2 2 2 2 2
(iii) (1– x ) (1 – y ) + 4xy = 1 – x – y + x y + 4xy (Note this step)
2 2 2 2
= 1 – x – y + x y + 2xy + 2xy
2 2 2 2
= (1 + 2xy + x y ) – (x – 2xy + y )
= (1 + xy) – (x – y) = (1 + xy + x – y) (1 + xy – x + y).
2 2
2 2 2 2
Ex. 13. Factorise: 2(ab – cd) + a + b – c – d .
Sol. 2(ab – cd) + a + b – c – d
2 2 2 2

= 2ab – 2cd + a2 + b2 – c2 – d2
Rearranging the terms in the expression,
= (a2 + 2ab + b2) – (c2 + 2cd + d2)
Ch 4–6 ICSE MATHEMATICS–IX

= (a + b)2 – (c + d)2
= {(a + b) + (c + d)} {(a + b) – (c + d)}
= (a + b + c + d) (a + b – c – d).
4 2 4 2 2 4
Ex. 14. Factorise: (i) x + 7x + 16 (ii) a + a b + b .
4 2 4 2 2
Sol. (i) x + 7x + 16 = x + 8x + 16 – x Adding and subtracting x2
= (x2 + 4)2 – (x)2
= (x2 + 4 + x) (x2 + 4 – x)
4 2 2 4 4 2 2 4 2 2
(ii) a + a b + b = (a + 2a b + b ) – a b Adding and subtracting a2b2
2 2 2 2
= (a + b ) – (ab)

d
= (a2 + b2 + ab) (a2 + b2 – ab).
2 2

ite
Ex. 15. Express (x + 6x – 7)(x – 6x + 7) as a difference of two squares.
2 2 2 2 2 2 2
Sol. (x + 6x – 7) (x – 6x + 7) = [x + (6x – 7)][x – (6x – 7)] = [(x ) – (6x – 7) ].

m
ExErCisE 4 (e)

Li
Factorise the following:
1. 7x − 7 2. 8 − 50y2z2 3. ab2 − ac2 4. 36x3 − x
2

ny
5. x(x − 1) + 7(x − 1) 6. t (t − 3) − (t − 3) 7. 5c2(c + 2)2 − 45(c 8. (a + b)2 − 1
2 2 2 2 2
45(c + 2)2

pa
9. 1 − (x − y) 10. 25a2 − 16(x − y)2 11. 20 − 45 (m 12. x4 − y4
2
n)2
(m + n)
13. x − 625 14. xy − yx 81x − 256y 16. a2 + ac + bc − b2
4 5 5 4 4
15. 81x 256y
om
17. 4a2 − b2 + 2a + b 18. x2 + 3x − y2 − 3y 19. a2 + b2 − 2ab − 4c2 20. 9x2 − 6xy + y2 − z2
21. x2 − 1 − 2a − a2 22. 4a2 + b2 − c2 + 4ab 23. x3 + 22x2 − x − 2 24. 1 + 2ab – (a2 + b2)
1
C

25. x2 + 2 – 11 26. x4 + 3x2 + 4


x
d

27. Factorise the following:


An

(i) 4a2 − b2 + 2a + b (ii) 9x2 − 4(yy + 2xx))2 ((iii) 9(x + y)2 − x2


22x) (ICSE)
ICSE 1992
28. Factorise : x − 3x − x + 3 ((ICSE 1992)
3 2
29. Factorise: (a2 – b2)(c2 – d 2) – 4abcd
30. Express (x + 8x + 15) (x − 8x 8x + 15) as a difference of two squares.
2 2
(ICSE)
n d

AnswErs
ha

1. 7(x + 1) (x − 1) 2. 2(2 + 5yz) (2 − 5yz) 3. a(b + c) (b − c)


C

(6x − 1) 5. (x + 1) (x − 1) (x + 7) 6. (t − 3) (t + 1) (t − 1)
2
4. x(6x + 1) (6x
S

7. 5(c + 2)2 ((cc + 3) ((c − 3) 8. (a + b − 1) (a + b + 1) 9. (1 + x − y) (1 − x + y)


10. (5a − 44xx + 44y
y (5a + 4x − 4y)
4y) 11. 5(2 − 3m − 3n) (2 + 3m + 3n) 12. (x − y) (x + y) (x2 + y2)
©

13. (x − 5) (x + 5) ((x2 + 25) 14. xy(y − x) (y + x) (y2 + x2)


15. (3x − 4y) (3x + 4y) (9x2 + 16y2) 16. (a + b) (a − b + c) 17. (2a + b) (2a − b + 1)
18. (x − y) (x + y + 3) 19. (a − b − 2c) (a − b + 2c) 20. (3x − y + z) (3x − y − z)
21. (x + 1 + a) (x − 1 − a) 22. (2a + b + c) (2a + b − c) 23. (x − 1) (x + 1) (x + 2)
1 1
24. (1 + a – b) (1 – a + b) 25. ( x - + 3)( x - - 3) 26. (x2 + 2 + x)(x2 + 2 – x)
x x

27. (i) (2a + b) (2a – b + 1) (ii) – (7x + 2y) (x + 2y) (iii) (4x + 3y) (2x + 3y)
28. (x + 1)(x – 1) (x – 3)
29. [a(c + d) + b(c – d)][a(c – d) – b (c + d)] 30. (x2 + 15)2 – (8x)2
FACTorISATIon Ch 4–7

Hints
24. 1 + 2ab – (a + b ) = 1 – (a + b – 2ab) 26. Add and subtract x2.
2 2 2 2

29. (a2 – b2)(c2 – d2) – 4abcd = a2c2 – b2c2 – a2d2 + b2d2 – 2abcd – 2abcd
= (a2c2 – 2abcd + b2d2) – (b2c2 + 2abcd + a2d2) = (ac – bd)2 – (bc + ad)2
Now factorise.
30. See Solved Ex. 15.

trinomials
Too find the factors of a non-perfect square trinomial of the second degree in which the coefficient of the
highest power is unity.

d
2
Type. x + bx + c

ite
Consider the product of (x + a) and (x + b), where a, b are any real numbers.
2
(x + a) (x + b) = x + (a + b)x + ab

m
))xx + ab can be factorised by finding
2
Reversing the above equation, we see that trinomials of the form x + (a + b)x
two factors of the constant term whose sum is the numerical coefficient of the middle term, and adding.

Li
Note. An expression of the form ax2 + bx + c is called a trinomial or a quadratic polynomial.

ny
Ex. 1. Factorise : x2 + 5x + 6.
Sol. The set of factors of 6 is {1, 2, 3, 6}. Thus, the constant term could be 6 × 1 or 2 × 3. Of these, the sum of
pa
one pair must give the coefficient 5 of the middle term. The sum of 2 and 3 is 5. Therefore, 2 and 3 are the second
om
terms of the binomials.
Since the trinomial is of the form +, +, +, the sign in each binomial is +. Hence the factors of
2
x + 5x + 6 are (x + 2) (x + 3).
C

2
Thus x + 5x + 6 = (x + 2) (x + 3).
Ex. 2. Factorise the following :
d

2
(i) x + 9x + 20 (ii) x2 − 7x + 12 (iii) x2 − 11x − 80 (iv) m2 + 17mn − 84n2.
An

Sol. (i) x + 9x + 20
2
2
= x + 5x + 4x + 20 (Find mentally two numbers whose sum is + 9 and product + 20.
d

2
= (x + 5x) + (4x + 20) The numbers are +5 and +4).
n

= x(x + 5) + 4(x + 5)
= (x + 5) (x + 4).
ha

(ii) x − 7x + 12
2
(Find two numbers whose product is +12 and sum −7. Both numbers
= x − 4xx − 33xx + 12
C

2
will be negative. They are −4 and −3).
= (x − 44x x)) − (3x
(3 − 12)
2
4x)
S

= x(x (x − 4) − 3 ((x − 4)
xx((x
= (x ( − 3).
(x − 4) (x
©

− 11x − 80
2
(iii) x (Since the product (−80) is to be negative, one number will be +ve
x − 16x + 5x − 80
2
= and the other −ve. Since the sum is to be −11, the greater number
(x − 16x) + (5x − 80)
2
= will be −ve, and the smaller +ve. the numbers are −16 and +5).
= x(x − 16) + 5(x − 16)
= (x − 16) (x + 5).

+ 17mn − 84n2
2
(iv) m
(Since the product (−84) is to be negative, one number will be +ve
= m2 + 21mn − 4mn − 84n2 and the other −ve. Since the sum is to be +17, the greater number
= (m2 + 21mn) − (4mn + 84n2) will be +ve, and the smaller −ve. the numbers are +21 and −4).
= m (m + 21n) − 4n (m + 21n)
= (m + 21n) (m − 4n).
Ch 4–8 ICSE MATHEMATICS–IX

Ex. 3. Factorise:
2 2 2
(i) (x + 5) (x – 10) + 14 (ii) 25x – (x – 36) (iii) (6 – x)2 – 3x
Sol. (i) (x + 5) (x – 10) + 14 = (x + 5x – 10x – 50) + 14
2

2 2
= x – 5x – 50 + 14 = x – 5x – 36
= x2 – 9x + 4x – 36 = (x2 – 9x) + (4x – 36)
= x(x – 9) + 4(x – 9) = (x – 9) (x + 4).
2 2 2
(ii) 25x – (x – 36) = (5x)2 – (x2 – 36)2
= (5x + x2 – 36) (5x – x2 + 36)
2 2
= (x + 5x – 36) (36 + 5x – x )

d
2 2
= (x + 9x – 4x – 36) (36 + 9x – 4x – x )

ite
= [x(x + 9) – 4(x + 9)] [9(4 + x) – x(4 + x)]
= [(x – 4) (x + 9)][(4 + x) (9 – x)]

m
= (x – 4)(x + 9) (4 + x) (9 – x).

Li
2
(iii) (6 – x) – 3x = (36 – 12 x + x2) – 3x
2 2
= x – 15x + 36 = x – 12x – 3x + 36

ny
= x(x – 12) – 3(x – 12) = (x – 12)(12)(xx – 3).
2 2
Ex. 4. Factorise: (i) 3y – 30y + 48 (ii) (p + q) + 2(p + q) – 3.
Sol. (i) 3y – 30y + 48 = 3(y – 10y + 16) = 3 (y – 8y – 22yy + 16)
2 2 2
pa First take out the common factor 3.
om
= 3[y(y – 8) – 2(y – 8)] = 3(y – 8)(
8)(yy – 2)
2
(ii) (p + q) + 2((p + q) – 3
C

2
Let (p + q) = x, then the expression becomes x + 22xx – 3.
2
Now, x + 2x – 3 = x2 + 3x – x – 3
d

= x(x + 3) – 1(
1(xx + 3) = ((x + 3)(x – 1)
An

(p + q + 3) (p
= (p ( + q – 1).

ExErCisE 4 (f )
n d

Factorise:
ha

1. a2 + 5a + 6 2. a2 + 6a + 8 3. p2 + 10p + 16 4. a2 + 13a + 42
5. a + 25a − 54 6. x2 + 5x − 176 7. y2 − 18y + 65 8. m2 − 29m + 204
2
C

9. b − 2b − 48 10. x2 − 11x − 102 11. 3 − 4t + t2 12. 51 − 20k + k2


2

13. 2x2 − 10 14. 3x3 − 33x2 + 84x 15. 5y2 − 45y − 110 16. x4 − 13x2 + 36
S

10xx + 12
17. x + 3xy 18. x4 – x2y2 – 72y4 19. a3b3 – 9a2b2 + 20ab 20. (x2 + x)2 + 4(x2 + x) – 21
2
88yy2
3xy – 88
©

AnswErs
1. (a + 3) (a + 2) 2. (a + 4) (a + 2) 3. (p + 8) (p + 2) 4. (a + 7) (a + 6)
5. (a + 27) (a − 2) 6. (x + 16) (x − 11) 7. (y − 13) (y − 5) 8. (m − 17) (m − 12)
9. (b − 8) (b + 6) 10. (x − 17) (x + 6) 11. (3 − t) (1 − t) 12. (3 − k) (17 − k)
13. 2(x − 2) (x − 3) 14. 3x(x − 7) (x − 4) 15. 5(y − 11) (y + 2)
16. (x + 2) (x – 2) (x + 3) (x – 3) 17. (x + 11y)(x – 8y)
18. 19. 20. (x2 + x + 7)(x2 + x – 3)
2 2
(x + 3y) (x – 3y) (x + 8y ) ab(ab – 5)(ab – 4)

Hints
16. Let x = y 19. Take ab common 20. Let x2 + x = y
2
FACTorISATIon Ch 4–9

Trinomials in which the coefficient of the highest power is not unity


Type : ax2 + bx + c
Study the following :
(3t + 5) (2t − 1) = 6t + 10t − 3t − 5 = 6t + 7t − 5
2 2

This shows that if we want to factorise 6t + 7t − 5, we should first write the middle term + 7t as 10t − 3t.
2

Note that (+10) + (−3) = +7 and (+10) (−3) = −30 = 6 × (−5) = (coefficient of t ) × (constant term). We therefore,
2

first multiply the coefficient of x and the constant term and get ac. Now, we find two numbers whose product is ac
2

and whose algebraic sum is equal to the coefficient of the middle term, that is, b.
Ex. 1. Factorise :
2
(i) 2x + 9x − 5 (ii) 12x2 − x − 1 (iii) 3 − 4x − 7x2.

d
Sol. (i) 2x + 9x − 5
2
Find two numbers whose product is 2 × (−(−5),
(−5), i.e., −10 and sum is
5), i.e.

ite
= 2x + 10x − x − 5
2
(+9). They are (+10) and (−1).
= 2x(x + 5) − 1(x + 5)

m
= (2x − 1) (x + 5)
(ii) 12x − x − 1 Find two numbers whose product is (+12) × (−1), i.e., −12 and sum is

Li
2

= 12x − 4x + 3x − 1
2 (−1). They are (−4)
4) and (+3).

ny
= 4x(3x − 1) + 1(3x − 1)
= (3x − 1) (4x + 1)
(iii) 3 − 4x − 7x
pa × 3, i.e., (−21) and sum is
2 Find two numbers whose product is (−7)
(
= −7x2 − 4x + 3 (−4). ((−7)
4). They are (−−7)
7) and (+3).
om
= −7x − 7x + 3x + 3
2

= (−7x − 7x) + (3x + 3)


2
C

= −7x (x + 1) + 3(x + 1)
= (x + 1) (−7x + 3)
d

= (x + 1) (3 − 7x)
An

Note. In (iii)) above we cannot change the signs of all terms to make the term of the highest degree, −7x , positive.
2
d

This is not possible in an expression. In an equation we can do so.


n

Ex. 2. Factorise : 2a2b2 + 5abc + 2c2.


ha

Sol. 2a2b2 + 55abc + 2c2 = 2a2b2 + 4abc + abc + 2c2


2a
(2a2b2 + 4abc) + (abc + 2c2)
C

= (2
= 2ab(ab + 2c) + c(ab + 2c) = (ab + 2c) (2ab + c).
S

3 2 2
Ex. 3. Factorise : 4x – 17x y – 15xy .
Sol. 4x3 – 17x2y – 15xy2 = x (4x2 – 17xy – 15y2)
©

= x(4x2 – 20xy + 3xy – 15y2) (Q – 20 × 3 = 60 and –20 + 3 = 17)


= x[4x(x – 5y) + 3y(x – 5y)]
= x[(x – 5y)(4x + 3y)] = x(x – 5y)(4x + 3y).
Ex. 4. Factorise :
(i) 8(x – 2y)2 + 2(x – 2y) – 15
(ii) 3(x + 1)2 – 10(x + 1)(y + 1) + 8(y + 1)2
Sol. (i) 8(x – 2y)2 + 2(x – 2y) – 15
Putting x – 2y = a, the expression reduces to
8a2 + 2a – 15 = 8a2 + 12a – 10a – 15
= 4a(2a + 3) – 5(2a + 3) = (2a + 3) (4a – 5)
Ch 4–10 ICSE MATHEMATICS–IX

= [2(x – 2y) + 3][4(x – 2y) – 5] [Putting a = x – 2y]


= (2x – 4y + 3) (4x – 8y – 5) .
(ii) 3(x + 1) – 10(x + 1)(y + 1) + 8(y + 1)2
2

Putting x + 1 = a and y + 1 = b, the above expression reduces to


3a2 – 10ab + 8b2 = 3a2 – 6ab – 4ab + 8b2
= 3a(a – 2b) – 4b (a – 2b) = (3a – 4b) (a – 2b)
= [3(x + 1) – 4(y + 1)][(x + 1) – 2(y + 1)]
= (3x + 3 – 4y – 4) (x + 1 – 2y – 2 ) = (3x – 4y – 1) (x – 2y – 1).
ExErCisE 4 (g)

d
Factorise :
1. 2x + 3x + 1 2. 3x2 + 17x + 10 3. 4. 6x
6x2 − 77x − 5
2
5x2 + 9x – 2

ite
5. 6y − 17y + 12 6. 8y2 − 2y − 1 7. 18b x2 + 18bx − 20b 8. 14 14xx2 − 60xy + 16y2
2

m
9. 30x2 + 103xy − 7y2 10. 12x2 − 29xy + 14y2 11. 15 + p(7 − 2p) 12. 2 − y(7 − 5y)
13. x(2x + 5) − 3 14. 1 − 18y − 63yy
2

Li
(ICSE)
15. 8a b – 10a b – 12ab (ICSE 1992) 16. 5(aa + b) – 3
3 2 2 3 2
2(a + b) – 5(

ny
AnswErs
1.
5.
(2x + 1) (x + 1)
(3y − 4) (2y − 3)
2. (3x + 2) (x + 5)
6. (4y + 1) (2y − 1)
3.
7. pa
(5x − 1) (x(x + 2)
(3xx − 2) (3x
4. (3x − 5) (2x + 1)
8. 2(7x − 2y) (x − 4y)
om
2b(3x
22bb(3 (3 + 5)
9. (2x + 7y) (15x − y) 10. (4x − 7y) (3x − 2y) 11. (5 − p)p) (3 + 2p)
2 12. (1 − y) (2 − 5y)
13. (x + 3) (2x − 1) 14. (1 + 3y) (1 – 21y) 15. 22ab (4 + 3b)(a – 2b) 16. (2a + 2b + 1) (a + b – 3)
ab(4
2ab(4a
C

Factorisation of sum or difference of cubes


d

In the last chapter, we found that, (a + bb))3 = a3 + b3 + 3ab (a + b)


An


3 3 3 2
a + b = (a + b) – 3ab ((aa + bb)) = ((a + b) [(a + b) – 3ab]
2 2 2 2
= (a + b) [a + b + 22ab – 3ab] = (a + b) (a + b – ab)
d

3 3
∴ a + b = (a + b)(a )(aa2 – ab + b2)
)(
n

3 3 3
Also, a – b = (a – bb)) + 3ab ab – b) = (a – b) [(a – b)2 + 3ab]
33ab(a
ha

2 2 2 2
= (a (a – 22ab + b + 3ab) = (a – b) (a + ab + b )
(a – bb)) (a
3 3
a – b = (a )( 2 + ab + b2)
(a – bb)(
b)(a
C

(a + b) and (a − ab + b ) on the right are called factors of a + b .


2 2 3 3

(a + ab + b ) on the right are called factors of a − b .


(a − b)) and (a
2 2 3 3
S

Help Box
©

3 3 2 2
(1st term) ± (2nd term) = (1st term ± 2nd term) [(1st term)  1st term × 2nd term + (2nd term) ]

3 3 1
Ex. 1. Factorise : (i) 8x + 27y (ii) y3 − 3
(iii) 8 (3x − 4y)3 − 125 (2x + y)3.
8z
Sol. (i) 8x + 27y = (2x) + (3y)
3 3 3 3

= (2x + 3y) [(2x)2 − (2x) (3y) + (3y)2]


2 2
= (2x + 3y) (4x − 6xy + 9y ).
3 2
1  1   1  2  1   1  
(ii) y − 3 = (y) −   =  y −  ( y ) + ( y )   +   
3 3
8z  2z   2 z    2 z   2 z  
 1  2 y 1 
=  y −  y + + 2
 2z   2z 4z 
FACTorISATIon Ch 4–11

(iii) 8(3x − 4y)3 − 125(2x + y)3


Let 3x − 4y = a and 2x + y = b. Then,
8(3x − 4y) − 125 (2x + y) = 8a − 125b
3 3 3 3

= (2a)3 − (5b)3 = (2a − 5b) (4a2 + 10ab + 25b2)


= {2(3x − 4y) −5 (2x + y)} {4(3x − 4y) + 10 (3x − 4y) (2x + y) + 25 (2x + y) }
2 2

= (6x − 8y − 10x − 5y)} {4(9x − 24xy + 16y ) + 10 (6x − 5xy − 4y ) + 25 (4x + 4xy + y )}
2 2 2 2 2 2

= (− 4x − 13y) (36x − 96xy + 64y + 60x − 50xy − 40y + 100x + 100xy + 25y )
2 2 2 2 2 2
2 2
= −(4x + 13y) (196x − 46xy + 49y ).
Ex. 2. Factorise: a6 − b6.
Sol. Method 1.

d
a − b = (a ) − (b ) = (a − b ) (a + b )
6 6 3 2 3 2 3 3 3 3

ite
= (a − b) (a + ab + b ) (a + b) (a − ab + b )
2 2 2 2

= (a − b) (a + b) (a + ab + b ) (a − ab + b ).
2 2 2 2

m
Method 2.

Li
a − b = (a ) − (b ) = (a − b ) [(a ) + (a ) (b ) + (b ) ]
6 6 2 3 2 3 2 2 2 2 2 2 2 2
Note this step. Adding
= (a − b) (a + b) (a + a b + b )
4 2 2 4 2 2

ny
and subtracting a b .
= (a − b) (a + b) [{(a ) + 2a b + (b ) } − a b ]
2 2 2 2 2 2 2 2

= (a − b) (a + b) [(a + b ) − (ab) ]
pa
2 2 2 2

= (a − b) (a + b) (a + b + ab) (a + b − ab)
2 2 2 2
ab)
om
= (a − b) (a + b) (a + ab + b ) (a − ab + b )
2 2 2 2

Clearly, Method 1 is simpler than Method 2. Thus, whenever it is possible to reduce a given expression as a
C

difference of two squares or as a difference of two cubes, always prefer the former to the latter.
Ex. 3. Factorise:
d

(i) a3 + b3 + a + b (ii) 250 xy3 – 16x4 (iii) 8a3 – b3 + 12a2b – 6ab2


An

1 2
(iv) (a3 + 3
+ 2a + )
a a
n d

Sol. (i) a + b + a + b = ((aa + b ) + ((a + b)


3 3 3 3
ha

)( 2 – ab + b2) + 1(a + b)
= ((aa + bb)(
b)(a
(a + b)(a2 – ab + b2 + 1).
= (a
C

(ii) xy3 – 16
250xy 16xx4 = 22x(125y3 – 8x3) = 2x[(5y)3 – (2x)3]
S

= 22x[(5y – 2x)(25y2 + (5y)(2x) + 4x2)]


= 2x(5y – 2x)((25y2 + 10xy + 4x2).
©

(iii) 8a3 – b3 + 12a2b – 6ab2 = {(2a)3 – (b)3} + 6ab(2a – b)


= {(2a – b) (4a2 + 2ab + b2)} + 6ab (2a – b)
= (2a – b){(4a2 + 2ab + b2) + 6ab}
= (2a – b)(4a2 + 8ab + b2).

1 2 1 1
(iv) a3 + 3
+ 2a + = {(a)3 + ( )3} +2(a + )
a a a a
1 1 1
= {(a + ) (a2 – 1 + 2 )} + 2 (a + )
a a a
Ch 4–12 ICSE MATHEMATICS–IX

 1   2 1  
=  a +   a – 1 + 2  + 2 
 a   a  
1 2 1
= (a + )(a + 1 + 2 ).
a a
6 3
Ex. 4. Factorise: a – 7a – 8
Sol. Putting a3 = x, the expression a6 – 7a3 – 8 becomes
x2 – 7x – 8 = x2 – 8x + x – 8
= x(x – 8) +1(x – 8) = (x – 8) (x + 1)
= (a3 – 8) (a3 + 1) = (a – 2)(a2 + 2a + 4) (a + 1) (a2 – a + 1).

d
ExErCisE 4 (h)

ite
Factorise the following :
1. a + 1 2. x3 + 8 3. 8x3 + 1 4. x3 − 27
3

m
5. a − 8 6. 27m3 − 8 7. x3 + 64 8. 8a3 − b6
3

Li
9. x6 + 8b3 10. 8a3 + 27b3 11. 27x3 − 8y3 12. 128x
128x3 + 2
1 3

ny
3
13. Factorise : 8 x − y . (ICSE) 14. 343x y + 512y 15. (2a
3 4
b)3 + (a + 2b)3
(2a + b)
27
16. 27(m + 2n)3 + (m − 6n)3

19. 64a − b
6 6
(ICSE 1992) 20. a3 – b3 + 4(a
pa
17. 8(a + b)3 − 27cc3 18. x6 − 1
3 1
21. x − 3 − 6 x +
6
om
4(a – bb))
x x
22. 64a3 + 125b3 + 12a2b + 15ab2 23. 375(
375(aa – bb))3 + 3 24. If a4 + b4 = a2b2, show that a6 + b6 = 0.
C
d

AnswErs
Answ
An

1. (a + 1) (a2 − a + 1) 2. ((xx + 2) ((x2 − 2x + 4) 3. (2x + 1) (4x2 − 2x + 1)


4. (x − 3) (x + 3x + 9) 5. ((aa − 2) (a2 + 2a + 4) 6. (3m − 2) (9m2 + 6m + 4)
2
d

7. (x + 4) (x − 4x + 16) 8. (2a − b2) (4a2 + 2ab2 + b4 ) 9. (x2 + 2b) (x4 − 2x2b + 4b2)
2
(2
n

10. 9b2) 11.


(2a + 3b) (4a − 6ab + 9b (3x − 2y) (9x2 + 6xy + 4y2) 12. 2 (4x + 1) (16x2 − 4x + 1)
2
ha

 y 2 2 y2 
13.  2 x −   4 x + xy +  14. y (7x + 8y) (49x2 − 56xy + 64y2) 15. 9(a + b) (a2 + ab + b2)
 3 3 9 
C

16. 4m(7m + 36mn 17. (2a + 2b − 3c) (4a2 + 4b2 + 9c2 + 8ab + 6ac + 6bc)
2
36mn + 108 108n2)
S

18. (x − 1) (x
(x + 1) ((xx + x + 1) (x − x + 1)
2 2

(2aa − b) (4a + 2ab + b ) (4a − 2ab + b ) 20. (a – b) (a + ab + b + 4)


19. (2a + bb)) (2
2 2 2 2 2 2
©

1 2 1
21. ( x − ) ( x − 5 + 2 ) 22. (4a + 5b) (15a – 17ab + 25b )
2 2
x x
23. 3 (5a – 5b + 1) (25a2 + 25b2 – 50ab – 5a + 5b + 1)

Hints
24. a + b = (a ) + (b )
6 6 2 3 2 3

= (a2 + b2) (a4 − a2b2 + b4) = (a2 + b2) [(a4 + b4) − a2b2].
= (a2 + b2) (a2b2 − a2b2) [Q a4 + b4 = a2b2 (given)]
= (a2 + b2) × 0 = 0
FACTorISATIon Ch 4–13

CHAptEr tEst
Factorise : (a) 1 (b) –1
1 2 1
2
1. 8x y – x 2. x + + 2 − 2x − (c) (d) 0
2 3 5
x 2 x 2
 1 1 2 1 
3. 2x – x – 6 4. a3 – 0.216
2
9. The product of  x −   x +   x + 2  is
 x x x 
5. 6x2y – xy – 2y 6. (x2 – 3x)2 – 8 (x2 – 3x) – 20
4 1 3 1
Multiple Choice Questions (a) x + 4
(b) x + −2
7. One of the factors of (x –1) – (x –1) is
2 x x3
1 1
(a) x2 – 1 (b) x + 1 x4 − 2
(d) x + +2

d
(c) 4
x x2

ite
(c) x – 1 (d) x + 4
x y 10. If x – 2y = 11 and xy = 8, then the value of
8. If + = −1 (x, y ≠ 0), then the value of x3 – 8y3 is

m
y x
(a) 1860 (b 1600
(b)
(b

Li
x3 – y3 is (c) 1859 (d) 2000

ny
AnswErs

1. x2 (2y – x) (4y2 + 2xy + x2)




1 
x 
1
x

2.  x +   x + − 2 
 pa 3. (x – 2) (2x + 3)
om
4. (a – 0.6) (a + 0.6a + 0.36) 5. y (3x – 2) (2x + 1) 6. (x – 1) (x – 2) (x + 2) (x – 5)
2

7. (c) 8. (d) 9. (c)


C

10. (c)
d
An
n d
ha
C
S
©
©
S
C
ha
nd
An
d
C
om
pa
ny
Li
m
ite
d

You might also like